Как найти исходную матрицу зная обратную

Матрица BB является обратной матрицей к квадратной матрице AA, если AB=BA=EAB = BA = E.
Из определения можно понять, что обратная матрица BB будет квадратной матрицей аналогичного порядка, какой имеет матрица AA (иначе какое-либо из произведений ABAB или BABA будет не определено).
Обратная матрица для исходной матрицы AA определяется так: A−1A^{-1}. Можно утверждать, что если A−1A^{-1} существует, то AA−1=A−1A=EAA^{-1} = A^{-1} A= E.
Также легко видеть, что (A−1)−1=A(A^{-1})^{-1} = A.

Если детерминант матрицы является нулем, то обратную к ней матрицу нельзя получить.

Онлайн-калькулятор

Квадратную матрицу AA можно назвать вырожденной матрицей тогда, когда определитель матрицы AA равен нулю, и невырожденной, если определитель не равен нулю.

Важно

В том случае, если обратная матрица может существовать, то она будет единственной.

Формула для вычисления обратной матрицы

Обратную матрицу A−1A^{-1} к матрице AA можно найти по формуле:

A−1=1det⁡A⋅A∗A^{-1}=frac{1}{det A}cdot A^*

det⁡Adet A — определитель матрицы A,A,

A∗A^* — транспонированая матрица алгебраических дополнений к матрице A.A.

Задача 1

Нужно найти обратную матрицу для следующей матрицы:

A=(1−20 342 −131)A = begin{pmatrix}
1& -2 & 0\
3 & 4 & 2\
-1& 3& 1 \
end{pmatrix}

Решение

Вычислим детерминант:

det⁡A=∣1−20342−131∣=1∣4231∣−(−2)∣32−11∣+0∣34−13∣=8det A = begin{vmatrix}
1 & -2 & 0 \
3 & 4 & 2 \
-1 & 3 & 1 \
end{vmatrix} = 1 begin{vmatrix}
4 & 2 \
3 & 1 \
end{vmatrix} — (-2) begin{vmatrix}
3 & 2 \
-1 & 1 \
end{vmatrix} +0 begin{vmatrix}
3 & 4 \
-1 & 3 \
end{vmatrix} = 8

Так как det⁡A≠0det A neq 0, то матрица – невырожденная, и обратная для нее существует.

Посчитаем алгебраические дополнение:

A11=(−1)1+1∣4231∣=−2,A_{11} = (-1)^{1+1} begin{vmatrix}
4 & 2 \
3 & 1 \
end{vmatrix} = -2,

A12=(−1)1+2∣32−11∣=−5,A_{12} = (-1)^{1+2} begin{vmatrix}
3 & 2 \
-1 & 1 \
end{vmatrix} = -5,

A13=(−1)1+3∣34−13∣=13A_{13} = (-1)^{1+3} begin{vmatrix}
3 & 4 \
-1 & 3 \
end{vmatrix} = 13
,

A21=(−1)2+1∣−2031∣=2A_{21} = (-1)^{2+1} begin{vmatrix}
-2 & 0 \
3 & 1 \
end{vmatrix} = 2
,

A22=(−1)2+2∣10−11∣=1A_{22} = (-1)^{2+2} begin{vmatrix}
1 & 0 \
-1 & 1 \
end{vmatrix} = 1
,

A23=(−1)2+3∣1−2−13∣=−1A_{23} = (-1)^{2+3} begin{vmatrix}
1 & -2 \
-1 & 3 \
end{vmatrix} = -1
,

A31=(−1)3+1∣−2042∣=−4A_{31} = (-1)^{3+1} begin{vmatrix}
-2 & 0 \
4 & 2 \
end{vmatrix} = -4
,

A32=(−1)3+2∣1032∣=−2A_{32} = (-1)^{3+2} begin{vmatrix}
1 & 0 \
3 & 2 \
end{vmatrix} = -2
,

A33=(−1)3+3∣1−234∣=10.A_{33} = (-1)^{3+3} begin{vmatrix}
1 & -2 \
3 & 4 \
end{vmatrix} = 10.

Обратная матрица:

A−1=18(−22−4−51−213−110)A^{-1} = frac{1}{8} begin{pmatrix}
-2 & 2 & -4 \
-5 & 1 & -2 \
13 & -1 & 10 \
end{pmatrix}

Важно

Чтобы избежать ошибок, необходимо сделать проверку: для этого нужно посчитать произведение первоначальной матрицы на конечную. Если в результате получится единичная матрица, то вы нашли обратную матрицу безошибочно.

Задача 2

Найдите обратную матрицу для матрицы:

A=(13−25)A = begin{pmatrix}
1 & 3\
-2 & 5 \
end{pmatrix}

Решение

det⁡A=11≠0→A−1det A= 11 neq 0 rightarrow A^{-1} – существует.

A11=(−1)1+1⋅5=5A_{11} = (-1)^ {1+1} cdot 5 = 5,

A12=(−1)1+2⋅(−2)=2A_{12} = (-1)^ {1+2} cdot (-2) = 2,

A21=(−1)2+1⋅3=−3A_{21} = (-1)^ {2+1} cdot 3 = -3,

A22=(−1)2+2⋅1=1.A_{22} = (-1)^ {2+2} cdot 1 = 1.

Ответ:

A−1=111(5−321)A^{-1} = frac{1}{11} begin{pmatrix}
5 & -3 \
2 & 1 \
end{pmatrix}

Нами был рассмотрен способ нахождения матрицы с помощью алгебраических дополнений. Существует еще один способ, который называется методом элементарных преобразований.

Метод элементарных преобразований

Метод основан на элементарных преобразованиях матриц, под которыми будем понимать такие преобразования, в результате которых сохраняется эквивалентность матриц:

  1. перестановка местами любых двух рядов (строк или столбцов) матрицы;
  2. умножение любого ряда матрицы (строки или столбца) на некоторое число, отличное от нуля;
  3. прибавление к любому ряду (строке или столбцу) матрицы другого ряда (строки или столбца), умноженного на некоторое число, отличное от нуля.

Рассмотрим алгоритм нахождения обратной матрицы данным методом.

Алгоритм нахождения обратной матрицы методом элементарных преобразований

  1. Из исходной матрицы AA и единичной матрицы EE того же порядка составить расширенную матрицу, т.е. матрицу вида (A∣E)begin{pmatrix}A|Eend{pmatrix}.
  2. С помощью элементарных преобразований над строками расширенной матрицы получить единичную матрицу слева от черты: (E∣A−1)begin{pmatrix}E|A^{-1}end{pmatrix}.
  3. Выписать обратную матрицу, которая находится справа от черты.
Задача 1

Найти матрицу K−1K^{-1}, если K=(1301)K=begin{pmatrix}1&3\0&1end{pmatrix}.

Из матрицы KK второго порядка и единичной матрицы второго порядка составим расширенную матрицу:

(1301∣1001)begin{pmatrix}left.begin{matrix}1&3\0&1end{matrix}right|begin{matrix}1&0\0&1end{matrix}end{pmatrix}.

Произведем элементарные преобразования расширенной матрицы.

Прибавим к строке №1 строку №3, умноженную на -3:

(1301∣1001)∼(1001∣1−301)begin{pmatrix}left.begin{matrix}1&3\0&1end{matrix}right|begin{matrix}1&0\0&1end{matrix}end{pmatrix}sim begin{pmatrix}left.begin{matrix}1&0\0&1end{matrix}right|begin{matrix}1&-3\0&1end{matrix}end{pmatrix}.

Слева получили единичную матрицу.

Выпишем обратную матрицу:

K−1=(1−301)K^{-1}=begin{pmatrix}1&-3\0&1end{pmatrix}.

Сделаем проверку, чтобы убедиться в том, что найденная матрица действительно является обратной.

K⋅K−1=(1301)⋅(1−301)=(1⋅1+3⋅01⋅(−3)+3⋅10⋅1+1⋅00⋅(−3)+1⋅1)=(1001)Kcdot K^{-1}=begin{pmatrix}1&3\0&1end{pmatrix}cdotbegin{pmatrix}1&-3\0&1end{pmatrix}=begin{pmatrix}1cdot1+3cdot0&1cdot(-3)+3cdot1\0cdot1+1cdot0&0cdot(-3)+1cdot1end{pmatrix}=begin{pmatrix}1&0\0&1end{pmatrix}.

Значит, обратная матрица найдена правильно.

Задача 2

Найти матрицу F−1F^{-1}, если F=(110010033)F=begin{pmatrix}1&1&0\0&1&0\0&3&3end{pmatrix}.

Из матрицы FF третьего порядка и единичной матрицы третьего порядка составим расширенную матрицу:

(110010033∣100010001)begin{pmatrix}left.begin{matrix}1&1&0\0&1&0\0&3&3end{matrix}right|begin{matrix}1&0&0\0&1&0\0&0&1end{matrix}end{pmatrix}.

Произведем элементарные преобразования расширенной матрицы.

Прибавим к строке №1 строку №2, умноженную на -1:

(110010033∣100010001)∼(100010033∣1−10010001)begin{pmatrix}left.begin{matrix}1&1&0\0&1&0\0&3&3end{matrix}right|begin{matrix}1&0&0\0&1&0\0&0&1end{matrix}end{pmatrix}sim begin{pmatrix}left.begin{matrix}1&0&0\0&1&0\0&3&3end{matrix}right|begin{matrix}1&-1&0\0&1&0\0&0&1end{matrix}end{pmatrix}.

Прибавим к строке №3 строку №2, умноженную на -3:

(100010033∣1−10010001)∼(100010003∣1−100100−31)begin{pmatrix}left.begin{matrix}1&0&0\0&1&0\0&3&3end{matrix}right|begin{matrix}1&-1&0\0&1&0\0&0&1end{matrix}end{pmatrix}sim begin{pmatrix}left.begin{matrix}1&0&0\0&1&0\0&0&3end{matrix}right|begin{matrix}1&-1&0\0&1&0\0&-3&1end{matrix}end{pmatrix}.

Умножим строку №3 на 13frac{1}{3}:

(100010003∣1−100100−31)∼(100010001∣1−100100−113)begin{pmatrix}left.begin{matrix}1&0&0\0&1&0\0&0&3end{matrix}right|begin{matrix}1&-1&0\0&1&0\0&-3&1end{matrix}end{pmatrix}sim begin{pmatrix}left.begin{matrix}1&0&0\0&1&0\0&0&1end{matrix}right|begin{matrix}1&-1&0\0&1&0\0&-1&frac{1}{3}end{matrix}end{pmatrix}.

Слева получили единичную матрицу.

Выпишем обратную матрицу:

F−1=(1−100100−113)F^{-1}=begin{pmatrix}1&-1&0\0&1&0\0&-1&frac{1}{3}end{pmatrix}.

Сделаем проверку, чтобы убедиться в том, что найденная матрица действительно является обратной.

F⋅F−1=(110010033)⋅(1−100100−113)=(100010001)Fcdot F^{-1}=begin{pmatrix}1&1&0\0&1&0\0&3&3end{pmatrix}cdotbegin{pmatrix}1&-1&0\0&1&0\0&-1&frac{1}{3}end{pmatrix}=begin{pmatrix}1&0&0\0&1&0\0&0&1end{pmatrix}.

Значит, обратная матрица найдена правильно.

Выполнение контрольных работ на заказ недорого от профильных авторов на бирже Студворк!

Нахождение
обратной матрицы.

В
этой статье разберемся с понятием
обратной матрицы, ее свойствами и
способами нахождения. Подробно остановимся
на решении примеров, в которых требуется
построить обратную матрицу для заданной.

Навигация
по странице.

    Обратная
    матрица — определение.

    Нахождение
    обратной матрицы с помощью матрицы из
    алгебраических дополнений.

    Свойства
    обратной матрицы.

    Нахождение
    обратной матрицы методом Гаусса-Жордана.

    Нахождение
    элементов обратной матрицы с помощью
    решения соответствующих систем линейных
    алгебраических уравнений.

Обратная
матрица — определение.

Понятие
обратной матрицы вводится лишь для
квадратных матриц, определитель которых
отличен от нуля, то есть для невырожденных
квадратных матриц.

Определение.

Матрица
называется
обратной для матрицы
,
определитель которой отличен от нуля ,
если справедливы равенства ,
где E

единичная матрица порядка n
на n
.

Нахождение
обратной матрицы с помощью матрицы из
алгебраических дополнений.

Как
же находить обратную матрицу для данной?

Во-первых,
нам потребуются понятия транспонированной
матрицы
,
минора матрицы и алгебраического
дополнения элемента матрицы.

Определение.

Минор
k-ого

порядка
матрицы A
порядка m
на n

это определитель матрицы порядка k
на k
,
которая получается из элементов
матрицы А
,
находящихся в выбранныхk
строках
и k
столбцах.
(k
не
превосходит наименьшего из чисел m
или n
).

Минор (n-1)-ого
порядка,
который составляется из элементов всех
строк, кроме i-ой
,
и всех столбцов, кроме j-ого
,
квадратной матрицы А
порядка n
на n
обозначим
как .

Иными
словами, минор получается
из квадратной матрицы А
порядка n
на n
вычеркиванием
элементов i-ой
строки
и j-ого
столбца.

Для
примера запишем, минор 2-ого
порядка,
который получаетсся из матрицы выбором
элементов ее второй, третьей строк и
первого, третьего столбцов .
Также покажем минор, который получается
из матрицы вычеркиванием
второй строки и третьего столбца .
Проиллюстрируем построение этих
миноров: и .

Определение.

Алгебраическим
дополнением
элемента квадратной
матрицы называют
минор (n-1)-ого
порядка,
который получается из матрицы А
,
вычеркиванием элементов ее i-ой
строки
и j-ого
столбца,
умноженный на .

Алгебраическое
дополнение элемента обозначается
как .
Таким обрзом, .

Например,
для матрицы алгебраическое
дополнение элемента есть .

Во-вторых,
нам пригодятся два свойства определителя,
которые мы разобрали в разделевычисление
определителя матрицы
:

На
основании этих свойств определителя,
определения операции
умножения матрицы на число
и
понятия обратной матрицы справедливо
равенство ,
где —
транспонированная матрица, элементами
которой являются алгебраические
дополнения .

Матрица действительно
является обратной для матрицы А
,
так как выполняются равенства .
Покажем это

Составим алгоритм
нахождения обратной матрицы
с
использованием равенства .

Разберем
алгоритм нахождения обратной матрицы
на примере.

Пример.

Дана
матрица .
Найдите обратную матрицу.

Решение.

Вычислим
определитель матрицы А
,
разложив его по элементам третьего
столбца:

Определитель
отличен от нуля, так что матрица А
обратима.

Найдем
матрицу из алгебраических дополнений:

Поэтому

Выполним
транспонирование матрицы из алгебраических
дополнений:

Теперь
находим обратную матрицу как :

Проверяем
полученный результат:


Равенства выполняются,
следовательно, обратная матрица найдена
верно.

Свойства
обратной матрицы.

Понятие
обратной матрицы, равенство ,
определения операций над матрицами и
свойства определителя матрицы позволяют
обосновать следующие свойства
обратной матрицы
:

Нахождение
элементов обратной матрицы с помощью
решения соответствующих систем линейных
алгебраических уравнений.

Рассмотрим
еще один способ нахождения обратной
матрицы для квадратной матрицы А
порядка n
на n
.

Этот
метод основан на решении n
систем
линейных неоднородных алгебраических
уравнений с n
неизвестными.
Неизвестными переменными в этих системах
уравнений являются элементы обратной
матрицы.

Идея
очень проста. Обозначим обратную матрицу
как X
,
то есть, .
Так как по определению обратной матрицы ,
то

Приравнивая
соответствующие элементы по столбцам,
получим n
систем
линейных уравнений

Решаем
их любым способом и из найденных значений
составляем обратную матрицу.

Разберем
этот метод на примере.

Пример.

Дана
матрица .
Найдите обратную матрицу.

Решение.

Примем .
Равенство дает
нам три системы линейных неоднородных
алгебраических уравнений:

Не
будем расписывать решение этих систем,
при необходимости обращайтесь к
разделурешение
систем линейных алгебраических уравнений
.

Из первой системы уравнений имеем ,
из второй — ,
из третьей — .
Следовательно, искомая обратная матрица
имеет вид .
Рекомендуем сделать проверку, чтобы
убедиться в правильности результата.

Подведем
итог.

Мы
рассмотрели понятие обратной матрицы,
ее свойства и три метода ее нахождения.

Пример
решений методом обратной матрицы

Задание
1.
Решить
СЛАУ методом обратной матрицы.
2
x 1 +
3x 2 +
3x 3 +
x 4 =
1
3
x 1 +
5x 2 +
3x 3 +
2x 4 =
2
5
x 1 +
7x 2 +
6x 3 +
2x 4 =
3
4
x 1 +
4x 2 +
3x 3 +
x 4 =
4

Начало
формы

Конец
формы

Решение
.
Запишем матрицу в виде:
Вектор
B:
B T =
(1,2,3,4)
Главный
определитель
Минор
для (1,1):
=
5 (6 1-3 2)-7 (3 1-3 2)+4 (3 2-6 2) = -3
Минор
для (2,1):
=
3 (6 1-3 2)-7 (3 1-3 1)+4 (3 2-6 1) = 0
Минор
для (3,1):
=
3 (3 1-3 2)-5 (3 1-3 1)+4 (3 2-3 1) = 3
Минор
для (4,1):
=
3 (3 2-6 2)-5 (3 2-6 1)+7 (3 2-3 1) =
3
Определитель
минора

= 2 (-3)-3 0+5 3-4 3 = -3

Транспонированная
матрица

Алгебраические
дополнения
∆ 1,1 =
5 (6 1-2 3)-3 (7 1-2 4)+2 (7 3-6 4) = -3
∆ 1,2 =
-3 (6 1-2 3)-3 (7 1-2 4)+1 (7 3-6 4) = 0
∆ 1,3 =
3 (3 1-2 3)-3 (5 1-2 4)+1 (5 3-3 4) = 3
∆ 1,4 =
-3 (3 2-2 6)-3 (5 2-2 7)+1 (5 6-3 7) = -3
∆ 2,1 =
-3 (6 1-2 3)-3 (5 1-2 4)+2 (5 3-6 4) = 9
∆ 2,2 =
2 (6 1-2 3)-3 (5 1-2 4)+1 (5 3-6 4) = 0
∆ 2,3 =
-2 (3 1-2 3)-3 (3 1-2 4)+1 (3 3-3 4) = -6
∆ 2,4 =
2 (3 2-2 6)-3 (3 2-2 5)+1 (3 6-3 5) = 3
∆ 3,1 =
3 (7 1-2 4)-5 (5 1-2 4)+2 (5 4-7 4) = -4
∆ 3,2 =
-2 (7 1-2 4)-3 (5 1-2 4)+1 (5 4-7 4) = 1
∆ 3,3 =
2 (5 1-2 4)-3 (3 1-2 4)+1 (3 4-5 4) = 1
∆ 3,4 =
-2 (5 2-2 7)-3 (3 2-2 5)+1 (3 7-5 5) = 0
∆ 4,1 =
-3 (7 3-6 4)-5 (5 3-6 4)+3 (5 4-7 4) =
-12
∆ 4,2 =
2 (7 3-6 4)-3 (5 3-6 4)+3 (5 4-7 4) = -3
∆ 4,3 =
-2 (5 3-3 4)-3 (3 3-3 4)+3 (3 4-5 4) = 9
∆ 4,4 =
2 (5 6-3 7)-3 (3 6-3 5)+3 (3 7-5 5) = -3
Обратная
матрица
Вектор
результатов X

X
= A -1 ∙
B

X T =
(2,-1,-0.33,1)
x 1 =
2
x 2 =
-1
x 3 =
-0.33
x 4 =
1

см.
также решений
СЛАУ методом обратной матрицы
online.
Для этого введите свои данные и получите
решение с подробными комментариями.

Задание
2
.
Систему уравнений записать в матричной
форме и решить ее с помощью обратной
матрицы. Сделать проверку полученного
решения.
Решение
:xml
:xls

Пример
2
.
Записать систему уравнений в матричной
форме и решить с помощью обратной
матрицы.
Решение
:xml
:xls

Пример
.
Дана система трех линейных уравнений
с тремя неизвестными. Требуется: 1) найти
ее решение с помощью формул
Крамера
;
2) записать систему в матричной форме и
решить ее средствами матричного
исчисления.
Методические
рекомендации
.
После решения методом Крамера, найдите
кнопку «Решение методом обратной
матрицы для исходных данных». Вы
получите соответствующее решение. Таким
образом, данные вновь заполнять не
придется.
Решение
.
Обозначим через А — матрицу коэффициентов
при неизвестных; X — матрицу-столбец
неизвестных; B — матрицу-столбец свободных
членов:

Вектор
B:
B T =(4,-3,-3)
С
учетом этих обозначений данная система
уравнений принимает следующую матричную
форму: А*Х = B.
Если матрица А —
невырожденная (ее определитель отличен
от нуля, то она имеет обратную матрицу
А -1 .
Умножив обе части уравнения на А -1 ,
получим: А -1 *А*Х
= А -1 *B,
А -1 *А=Е.
Это
равенство называется матричной
записью решения системы линейных
уравнений
.
Для нахождения решения системы уравнений
необходимо вычислить обратную матрицу
А -1 .
Система
будет иметь решение, если определитель
матрицы A отличен от нуля.
Найдем
главный
определитель.
∆=-1 (-2 (-1)-1 1)-3 (3 (-1)-1 0)+2 (3 1-(-2 0))=14
Итак,
определитель 14 ≠ 0, поэтому продолжаем
решение. Для этого найдем обратную
матрицу через алгебраические
дополнения.
Пусть имеем невырожденную
матрицу А:

Вычисляем
алгебраические дополнения.

∆ 1,1 =(-2 (-1)-1 1)=1

∆ 1,2 =-(3 (-1)-0 1)=3

∆ 1,3 =(3 1-0 (-2))=3

∆ 2,1 =-(3 (-1)-1 2)=5

∆ 2,2 =(-1 (-1)-0 2)=1

∆ 2,3 =-(-1 1-0 3)=1

∆ 3,1 =(3 1-(-2 2))=7

∆ 3,2 =-(-1 1-3 2)=7

X T =(-1,1,2)
x 1 = -14 / 14 =-1
x 2 = 14 / 14 =1
x 3 = 28 / 14 =2
Проверка
.
-1 -1+3 1+0 2=4
3 -1+-2 1+1 2=-3
2 -1+1 1+-1 2=-3
doc
:xml
:xls

Ответ:
-1,1,2.

Похожие на обратные по многим свойствам.

Энциклопедичный YouTube

    1
    /
    5

    ✪ Как находить обратную матрицу — bezbotvy

    ✪ Обратная матрица (2 способа нахождения)

    ✪ Обратная матрица #1

    ✪ 2015-01-28. Обратная матрица 3×3

    ✪ 2015-01-27. Обратная матрица 2х2

    Субтитры


Свойства обратной матрицы


  • det
    A

    1
    =
    1
    det
    A
    {displaystyle det A^{-1}={frac {1}{det A}}}
    , где
    det
    {displaystyle det }
    обозначает определитель .

  • (A
    B)

    1
    =
    B

    1
    A

    1
    {displaystyle (AB)^{-1}=B^{-1}A^{-1}}
    для двух квадратных обратимых матриц
    A
    {displaystyle A}

    и
    B
    {displaystyle B}

    .


  • (A
    T)

    1
    =
    (A

    1)
    T
    {displaystyle (A^{T})^{-1}=(A^{-1})^{T}}
    , где
    (.
    .
    .)
    T
    {displaystyle (…)^{T}}
    обозначает транспонированную матрицу.

  • (k
    A)

    1
    =
    k

    1
    A

    1
    {displaystyle (kA)^{-1}=k^{-1}A^{-1}}
    для любого коэффициента
    k

    0
    {displaystyle knot =0}
    .

  • E

    1
    =
    E
    {displaystyle E^{-1}=E}
    .
  • Если необходимо решить систему линейных уравнений , (b — ненулевой вектор) где
    x
    {displaystyle x}

    — искомый вектор, и если
    A

    1
    {displaystyle A^{-1}}
    существует, то
    x
    =
    A

    1
    b
    {displaystyle x=A^{-1}b}
    . В противном случае либо размерность пространства решений больше нуля, либо их нет вовсе.

Способы нахождения обратной матрицы

Если матрица обратима, то для нахождения обратной матрицы можно воспользоваться одним из следующих способов:

Точные (прямые) методы

Метод Гаусса-Жордана

Возьмём две матрицы: саму A
и единичную E
. Приведём матрицу A
к единичной матрице методом Гаусса-Жордана применяя преобразования по строкам (можно также применять преобразования и по столбцам, но не в перемешку). После применения каждой операции к первой матрице применим ту же операцию ко второй. Когда приведение первой матрицы к единичному виду будет завершено, вторая матрица окажется равной A −1
.

При использовании метода Гаусса первая матрица будет умножаться слева на одну из элементарных матриц
Λ
i
{displaystyle Lambda _{i}}
(трансвекцию или диагональную матрицу с единицами на главной диагонали, кроме одной позиции):


Λ
1



Λ
n

A
=
Λ
A
=
E

Λ
=
A

1
{displaystyle Lambda _{1}cdot dots cdot Lambda _{n}cdot A=Lambda A=ERightarrow Lambda =A^{-1}}
.

Λ
m
=
[
1

0

a
1
m
/
a
m
m
0

0

0

1

a
m

1
m
/
a
m
m
0

0
0

0
1
/
a
m
m
0

0
0

0

a
m
+
1
m
/
a
m
m
1

0

0

0

a
n
m
/
a
m
m
0

1
]
{displaystyle Lambda _{m}={begin{bmatrix}1&dots &0&-a_{1m}/a_{mm}&0&dots &0\&&&dots &&&\0&dots &1&-a_{m-1m}/a_{mm}&0&dots &0\0&dots &0&1/a_{mm}&0&dots &0\0&dots &0&-a_{m+1m}/a_{mm}&1&dots &0\&&&dots &&&\0&dots &0&-a_{nm}/a_{mm}&0&dots &1end{bmatrix}}}
.

Вторая матрица после применения всех операций станет равна
Λ
{displaystyle Lambda }
, то есть будет искомой. Сложность алгоритма —
O
(n
3)
{displaystyle O(n^{3})}
.

С помощью матрицы алгебраических дополнений

Матрица, обратная матрице
A
{displaystyle A}

, представима в виде

A

1
=
adj
(A)
det
(A)
{displaystyle {A}^{-1}={{{mbox{adj}}(A)} over {det(A)}}}

где
adj
(A)
{displaystyle {mbox{adj}}(A)}
— присоединенная матрица ;

Сложность алгоритма зависит от сложности алгоритма расчета определителя O det и равна O(n²)·O det .

Использование LU/LUP-разложения

Матричное уравнение
A
X
=
I
n
{displaystyle AX=I_{n}}
для обратной матрицы
X
{displaystyle X}

можно рассматривать как совокупность
n
{displaystyle n}

систем вида
A
x
=
b
{displaystyle Ax=b}
. Обозначим
i
{displaystyle i}

-ый столбец матрицы
X
{displaystyle X}

через
X
i
{displaystyle X_{i}}

; тогда
A
X
i
=
e
i
{displaystyle AX_{i}=e_{i}}
,
i
=
1
,

,
n
{displaystyle i=1,ldots ,n}
,поскольку
i
{displaystyle i}

-м столбцом матрицы
I
n
{displaystyle I_{n}}

является единичный вектор
e
i
{displaystyle e_{i}}

. другими словами, нахождение обратной матрицы сводится к решению n уравнений с одной матрицей и разными правыми частями. После выполнения LUP-разложения (время O(n³)) на решение каждого из n уравнений нужно время O(n²), так что и эта часть работы требует времени O(n³) .

Если матрица A невырождена, то для неё можно рассчитать LUP-разложение
P
A
=
L
U
{displaystyle PA=LU}
. Пусть
P
A
=
B
{displaystyle PA=B}
,
B

1
=
D
{displaystyle B^{-1}=D}
. Тогда из свойств обратной матрицы можно записать:
D
=
U

1
L

1
{displaystyle D=U^{-1}L^{-1}}
. Если умножить это равенство на U и L то можно получить два равенства вида
U
D
=
L

1
{displaystyle UD=L^{-1}}
и
D
L
=
U

1
{displaystyle DL=U^{-1}}
. Первое из этих равенств представляет собой систему из n² линейных уравнений для
n
(n
+
1)
2
{displaystyle {frac {n(n+1)}{2}}}
из которых известны правые части (из свойств треугольных матриц). Второе представляет также систему из n² линейных уравнений для
n
(n

1)
2
{displaystyle {frac {n(n-1)}{2}}}
из которых известны правые части (также из свойств треугольных матриц). Вместе они представляют собой систему из n² равенств. С помощью этих равенств можно реккурентно определить все n² элементов матрицы D. Тогда из равенства (PA) −1 = A −1 P −1 = B −1 = D. получаем равенство
A

1
=
D
P
{displaystyle A^{-1}=DP}
.

В случае использования LU-разложения не требуется перестановки столбцов матрицы D но решение может разойтись даже если матрица A невырождена.

Сложность алгоритма — O(n³).

Итерационные методы

Методы Шульца

{
Ψ
k
=
E

A
U
k
,
U
k
+
1
=
U
k

i
=
0
n
Ψ
k
i
{displaystyle {begin{cases}Psi _{k}=E-AU_{k},\U_{k+1}=U_{k}sum _{i=0}^{n}Psi _{k}^{i}end{cases}}}

Оценка погрешности

Выбор начального приближения

Проблема выбора начального приближения в рассматриваемых здесь процессах итерационного обращения матриц не позволяет относиться к ним как к самостоятельным универсальным методам, конкурирующими с прямыми методами обращения, основанными, например, на LU-разложении матриц. Имеются некоторые рекомендации по выбору
U
0
{displaystyle U_{0}}

, обеспечивающие выполнение условия
ρ

0)
<
1
{displaystyle rho (Psi _{0})<1}
(спектральный радиус матрицы меньше единицы), являющегося необходимым и достаточным для сходимости процесса. Однако при этом, во-первых, требуется знать сверху оценку спектра обращаемой матрицы A либо матрицы
A
A
T
{displaystyle AA^{T}}
(а именно, если A — симметричная положительно определённая матрица и
ρ
(A)

β
{displaystyle rho (A)leq beta }
, то можно взять
U
0
=
α
E
{displaystyle U_{0}={alpha }E}
, где ; если же A — произвольная невырожденная матрица и
ρ
(A
A
T)

β
{displaystyle rho (AA^{T})leq beta }
, то полагают
U
0
=
α
A
T
{displaystyle U_{0}={alpha }A^{T}}
, где также
α

(0
,
2
β)
{displaystyle alpha in left(0,{frac {2}{beta }}right)}
; можно конечно упростить ситуацию и, воспользовавшись тем, что
ρ
(A
A
T)

k
A
A
T
k
{displaystyle rho (AA^{T})leq {mathcal {k}}AA^{T}{mathcal {k}}}
, положить
U
0
=
A
T

A
A
T

{displaystyle U_{0}={frac {A^{T}}{|AA^{T}|}}}
). Во-вторых, при таком задании начальной матрицы нет гарантии, что

Ψ
0

{displaystyle |Psi _{0}|}
будет малой (возможно, даже окажется

Ψ
0

>
1
{displaystyle |Psi _{0}|>1}
), и высокий порядок скорости сходимости обнаружится далеко не сразу.

Примеры

Матрица 2х2


A

1
=
[
a
b
c
d
]

1
=
1
det
(A)
[
d

b

c
a
]
=
1
a
d

b
c
[
d

b

c
a
]
.
{displaystyle mathbf {A} ^{-1}={begin{bmatrix}a&b\c&d\end{bmatrix}}^{-1}={frac {1}{det(mathbf {A})}}{begin{bmatrix},,,d&!!-b\-c&,a\end{bmatrix}}={frac {1}{ad-bc}}{begin{bmatrix},,,d&!!-b\-c&,a\end{bmatrix}}.}

Обращение матрицы 2х2 возможно только при условии, что
a
d

b
c
=
det
A

0
{displaystyle ad-bc=det Aneq 0}
.

Эта тема является одной из самых ненавистных среди студентов. Хуже, наверное, только определители.

Фишка в том, что само понятие обратного элемента (и я сейчас не только о матрицах) отсылает нас к операции умножения. Даже в школьной программе умножение считается сложной операцией, а уж умножение матриц — вообще отдельная тема, которой у меня посвящён целый параграф и видеоурок.

Сегодня мы не будем вдаваться в подробности матричных вычислений. Просто вспомним: как обозначаются матрицы, как они умножаются и что из этого следует.

Повторение: умножение матриц

Прежде всего договоримся об обозначениях. Матрицей $A$ размера $left[ mtimes n right]$ называется просто таблица из чисел, в которой ровно $m$ строк и $n$ столбцов:

=underbrace{left[ begin{matrix} {{a}_{11}} & {{a}_{12}} & … & {{a}_{1n}} \ {{a}_{21}} & {{a}_{22}} & … & {{a}_{2n}} \ … & … & … & … \ {{a}_{m1}} & {{a}_{m2}} & … & {{a}_{mn}} \end{matrix} right]}_{n}]

Чтобы случайно не перепутать строки и столбцы местами (поверьте, на экзамене можно и единицу с двойкой перепутать — что уж говорить про какие-то там строки), просто взгляните на картинку:


Определение индексов для клеток матрицы

Что происходит? Если разместить стандартную систему координат $OXY$ в левом верхнем углу и направить оси так, чтобы они охватывали всю матрицу, то каждой клетке этой матрицы можно однозначно сопоставить координаты $left(x;y right)$ — это и будет номер строки и номер столбца.

Почему система координат размещена именно в левом верхнем углу? Да потому что именно оттуда мы начинаем читать любые тексты. Это очень просто запомнить.

А почему ось $x$ направлена именно вниз, а не вправо? Опять всё просто: возьмите стандартную систему координат (ось $x$ идёт вправо, ось $y$ — вверх) и поверните её так, чтобы она охватывала матрицу. Это поворот на 90 градусов по часовой стрелке — его результат мы и видим на картинке.

В общем, как определять индексы у элементов матрицы, мы разобрались. Теперь давайте разберёмся с умножением.

Определение. Матрицы $A=left[ mtimes n right]$ и $B=left[ ntimes k right]$, когда количество столбцов в первой совпадает с количеством строк во второй, называются согласованными.

Именно в таком порядке. Можно сумничать и сказать, мол, матрицы $A$ и $B$ образуют упорядоченную пару $left(A;B right)$: если они согласованы в таком порядке, то совершенно необязательно, что $B$ и $A$, т.е. пара $left(B;A right)$ — тоже согласована.

Умножать можно только согласованные матрицы.

Определение. Произведение согласованных матриц $A=left[ mtimes n right]$ и $B=left[ ntimes k right]$ — это новая матрица $C=left[ mtimes k right]$, элементы которой ${{c}_{ij}}$ считаются по формуле:

[{{c}_{ij}}=sumlimits_{k=1}^{n}{{{a}_{ik}}}cdot {{b}_{kj}}]

Другими словами: чтобы получить элемент ${{c}_{ij}}$ матрицы $C=Acdot B$, нужно взять $i$-строку первой матрицы, $j$-й столбец второй матрицы, а затем попарно перемножить элементы из этой строки и столбца. Результаты сложить.

Да, вот такое суровое определение. Из него сразу следует несколько фактов:

  1. Умножение матриц, вообще говоря, некоммутативно: $Acdot Bne Bcdot A$;
  2. Однако умножение ассоциативно: $left(Acdot B right)cdot C=Acdot left(Bcdot C right)$;
  3. И даже дистрибутивно: $left(A+B right)cdot C=Acdot C+Bcdot C$;
  4. И ещё раз дистрибутивно: $Acdot left(B+C right)=Acdot B+Acdot C$.

Дистрибутивность умножения пришлось отдельно описывать для левого и правого множителя-суммы как раз из-за некоммутативности операции умножения.

Если всё же получается так, что $Acdot B=Bcdot A$, такие матрицы называются перестановочными.

Среди всех матриц, которые там на что-то умножаются, есть особые — те, которые при умножении на любую матрицу $A$ снова дают $A$:

Определение. Матрица $E$ называется единичной, если $Acdot E=A$ или $Ecdot A=A$. В случае с квадратной матрицей $A$ можем записать:

Единичная матрица — частый гость при решении матричных уравнений. И вообще частый гость в мире матриц.:)

А ещё из-за этой $E$ кое-кто придумал всю ту дичь, которая будет написана дальше.

Что такое обратная матрица

Поскольку умножение матриц — весьма трудоёмкая операция (приходится перемножать кучу строчек и столбцов), то понятие обратной матрицы тоже оказывается не самым тривиальным. И требующим некоторых пояснений.

Ключевое определение

Что ж, пора познать истину.

Определение. Матрица $B$ называется обратной к матрице $A$ , если

Обратная матрица обозначается через ${{A}^{-1}}$ (не путать со степенью!), поэтому определение можно переписать так:

Казалось бы, всё предельно просто и ясно. Но при анализе такого определения сразу возникает несколько вопросов:

  1. Всегда ли существует обратная матрица? И если не всегда, то как определить: когда она существует, а когда — нет?
  2. А кто сказал, что такая матрица ровно одна? Вдруг для некоторой исходной матрицы $A$ найдётся целая толпа обратных?
  3. Как выглядят все эти «обратные»? И как, собственно, их считать?

Насчёт алгоритмов вычисления — об этом мы поговорим чуть позже. Но на остальные вопросы ответим прямо сейчас. Оформим их в виде отдельных утверждений-лемм.

Основные свойства

Начнём с того, как в принципе должна выглядеть матрица $A$, чтобы для неё существовала ${{A}^{-1}}$. Сейчас мы убедимся в том, что обе эти матрицы должны быть квадратными, причём одного размера: $left[ ntimes n right]$.

Лемма 1 . Дана матрица $A$ и обратная ей ${{A}^{-1}}$. Тогда обе эти матрицы — квадратные, причём одинакового порядка $n$.

Доказательство. Всё просто. Пусть матрица $A=left[ mtimes n right]$, ${{A}^{-1}}=left[ atimes b right]$. Поскольку произведение $Acdot {{A}^{-1}}=E$ по определению существует, матрицы $A$ и ${{A}^{-1}}$ согласованы в указанном порядке:

[begin{align} & left[ mtimes n right]cdot left[ atimes b right]=left[ mtimes b right] \ & n=a end{align}]

Это прямое следствие из алгоритма перемножения матриц: коэффициенты $n$ и $a$ являются «транзитными» и должны быть равны.

Вместе с тем определено и обратное умножение: ${{A}^{-1}}cdot A=E$, поэтому матрицы ${{A}^{-1}}$ и $A$ тоже согласованы в указанном порядке:

[begin{align} & left[ atimes b right]cdot left[ mtimes n right]=left[ atimes n right] \ & b=m end{align}]

Таким образом, без ограничения общности можем считать, что $A=left[ mtimes n right]$, ${{A}^{-1}}=left[ ntimes m right]$. Однако согласно определению $Acdot {{A}^{-1}}={{A}^{-1}}cdot A$, поэтому размеры матриц строго совпадают:

[begin{align} & left[ mtimes n right]=left[ ntimes m right] \ & m=n end{align}]

Вот и получается, что все три матрицы — $A$, ${{A}^{-1}}$ и $E$ — являются квадратными размером $left[ ntimes n right]$. Лемма доказана.

Что ж, уже неплохо. Мы видим, что обратимыми бывают лишь квадратные матрицы. Теперь давайте убедимся, что обратная матрица всегда одна.

Лемма 2 . Дана матрица $A$ и обратная ей ${{A}^{-1}}$. Тогда эта обратная матрица — единственная.

Доказательство. Пойдём от противного: пусть у матрицы $A$ есть хотя бы два экземпляра обратных —$B$ и $C$. Тогда, согласно определению, верны следующие равенства:

[begin{align} & Acdot B=Bcdot A=E; \ & Acdot C=Ccdot A=E. \ end{align}]

Из леммы 1 мы заключаем, что все четыре матрицы — $A$, $B$, $C$ и $E$ — являются квадратными одинакового порядка: $left[ ntimes n right]$. Следовательно, определено произведение:

Поскольку умножение матриц ассоциативно (но не коммутативно!), мы можем записать:

[begin{align} & Bcdot Acdot C=left(Bcdot A right)cdot C=Ecdot C=C; \ & Bcdot Acdot C=Bcdot left(Acdot C right)=Bcdot E=B; \ & Bcdot Acdot C=C=BRightarrow B=C. \ end{align}]

Получили единственно возможный вариант: два экземпляра обратной матрицы равны. Лемма доказана.

Приведённые рассуждения почти дословно повторяют доказательство единственность обратного элемента для всех действительных чисел $bne 0$. Единственное существенное дополнение — учёт размерности матриц.

Впрочем, мы до сих пор ничего не знаем о том, всякая ли квадратная матрица является обратимой. Тут нам на помощь приходит определитель — это ключевая характеристика для всех квадратных матриц.

Лемма 3 . Дана матрица $A$. Если обратная к ней матрица ${{A}^{-1}}$ существует, то определитель исходной матрицы отличен от нуля:

[left| A right|ne 0]

Доказательство. Мы уже знаем, что $A$ и ${{A}^{-1}}$ — квадратные матрицы размера $left[ ntimes n right]$. Следовательно, для каждой из них можно вычислить определитель: $left| A right|$ и $left| {{A}^{-1}} right|$. Однако определитель произведения равен произведению определителей:

[left| Acdot B right|=left| A right|cdot left| B right|Rightarrow left| Acdot {{A}^{-1}} right|=left| A right|cdot left| {{A}^{-1}} right|]

Но согласно определению $Acdot {{A}^{-1}}=E$, а определитель $E$ всегда равен 1, поэтому

[begin{align} & Acdot {{A}^{-1}}=E; \ & left| Acdot {{A}^{-1}} right|=left| E right|; \ & left| A right|cdot left| {{A}^{-1}} right|=1. \ end{align}]

Произведение двух чисел равно единице только в том случае, когда каждое из этих чисел отлично от нуля:

[left| A right|ne 0;quad left| {{A}^{-1}} right|ne 0.]

Вот и получается, что $left| A right|ne 0$. Лемма доказана.

На самом деле это требование вполне логично. Сейчас мы разберём алгоритм нахождения обратной матрицы — и станет совершенно ясно, почему при нулевом определителе никакой обратной матрицы в принципе не может существовать.

Но для начала сформулируем «вспомогательное» определение:

Определение. Вырожденная матрица — это квадратная матрица размера $left[ ntimes n right]$, чей определитель равен нулю.

Таким образом, мы можем утверждать, что всякая обратимая матрица является невырожденной.

Как найти обратную матрицу

Сейчас мы рассмотрим универсальный алгоритм нахождения обратных матриц. Вообще, существует два общепринятых алгоритма, и второй мы тоже сегодня рассмотрим.

Тот, который будет рассмотрен сейчас, очень эффективен для матриц размера $left[ 2times 2 right]$ и — частично — размера $left[ 3times 3 right]$. А вот начиная с размера $left[ 4times 4 right]$ его лучше не применять. Почему — сейчас сами всё поймёте.

Алгебраические дополнения

Готовьтесь. Сейчас будет боль. Нет, не переживайте: к вам не идёт красивая медсестра в юбке, чулках с кружевами и не сделает укол в ягодицу. Всё куда прозаичнее: к вам идут алгебраические дополнения и Её Величество «Союзная Матрица».

Начнём с главного. Пусть имеется квадратная матрица размера $A=left[ ntimes n right]$, элементы которой именуются ${{a}_{ij}}$. Тогда для каждого такого элемента можно определить алгебраическое дополнение:

Определение. Алгебраическое дополнение ${{A}_{ij}}$ к элементу ${{a}_{ij}}$, стоящего в $i$-й строке и $j$-м столбце матрицы $A=left[ ntimes n right]$ — это конструкция вида

[{{A}_{ij}}={{left(-1 right)}^{i+j}}cdot M_{ij}^{*}]

Где $M_{ij}^{*}$ — определитель матрицы, полученной из исходной $A$ вычёркиванием той самой $i$-й строки и $j$-го столбца.

Ещё раз. Алгебраическое дополнение к элементу матрицы с координатами $left(i;j right)$ обозначается как ${{A}_{ij}}$ и считается по схеме:

  1. Сначала вычёркиваем из исходной матрицы $i$-строчку и $j$-й столбец. Получим новую квадратную матрицу, и её определитель мы обозначаем как $M_{ij}^{*}$.
  2. Затем умножаем этот определитель на ${{left(-1 right)}^{i+j}}$ — поначалу это выражение может показаться мозговыносящим, но по сути мы просто выясняем знак перед $M_{ij}^{*}$.
  3. Считаем — получаем конкретное число. Т.е. алгебраическое дополнение — это именно число, а не какая-то новая матрица и т.д.

Саму матрицу $M_{ij}^{*}$ называют дополнительным минором к элементу ${{a}_{ij}}$. И в этом смысле приведённое выше определение алгебраического дополнения является частным случаем более сложного определения — того, что мы рассматривали в уроке про определитель.

Важное замечание. Вообще-то во «взрослой» математике алгебраические дополнения определяются так:

  1. Берём в квадратной матрице $k$ строчек и $k$ столбцов. На их пересечении получится матрица размера $left[ ktimes k right]$ — её определитель называется минором порядка $k$ и обозначается ${{M}_{k}}$.
  2. Затем вычёркиваем эти «избранные» $k$ строчек и $k$ столбцов. Снова получится квадратная матрица — её определитель называется дополнительным минором и обозначается $M_{k}^{*}$.
  3. Умножаем $M_{k}^{*}$ на ${{left(-1 right)}^{t}}$, где $t$ — это (вот сейчас внимание!) сумма номеров всех выбранных строчек и столбцов. Это и будет алгебраическое дополнение.

Взгляните на третий шаг: там вообще-то сумма $2k$ слагаемых! Другое дело, что для $k=1$ мы получим лишь 2 слагаемых — это и будут те самые $i+j$ — «координаты» элемента ${{a}_{ij}}$, для которого мы ищем алгебраическое дополнение.

Таким образом сегодня мы используем слегка упрощённое определение. Но как мы увидим в дальнейшем, его окажется более чем достаточно. Куда важнее следующая штука:

Определение. Союзная матрица $S$ к квадратной матрице $A=left[ ntimes n right]$ — это новая матрица размера $left[ ntimes n right]$, которая получается из $A$ заменой ${{a}_{ij}}$ алгебраическими дополнениями ${{A}_{ij}}$:

\Rightarrow S=left[ begin{matrix} {{A}_{11}} & {{A}_{12}} & … & {{A}_{1n}} \ {{A}_{21}} & {{A}_{22}} & … & {{A}_{2n}} \ … & … & … & … \ {{A}_{n1}} & {{A}_{n2}} & … & {{A}_{nn}} \end{matrix} right]]

Первая мысль, возникающая в момент осознания этого определения — «это сколько же придётся всего считать!» Расслабьтесь: считать придётся, но не так уж и много.:)

Что ж, всё это очень мило, но зачем это нужно? А вот зачем.

Основная теорема

Вернёмся немного назад. Помните, в Лемме 3 утверждалось, что обратимая матрица $A$ всегда не вырождена (т.е. её определитель отличен от нуля: $left| A right|ne 0$).

Так вот, верно и обратное: если матрица $A$ не вырождена, то она всегда обратима. И даже существует схема поиска ${{A}^{-1}}$. Зацените:

Теорема об обратной матрице. Пусть дана квадратная матрица $A=left[ ntimes n right]$, причём её определитель отличен от нуля: $left| A right|ne 0$. Тогда обратная матрица ${{A}^{-1}}$ существует и считается по формуле:

[{{A}^{-1}}=frac{1}{left| A right|}cdot {{S}^{T}}]

А теперь — всё то же самое, но разборчивым почерком. Чтобы найти обратную матрицу, нужно:

  1. Посчитать определитель $left| A right|$ и убедиться, что он отличен от нуля.
  2. Составить союзную матрицу $S$, т.е. посчитать 100500 алгебраических дополнений ${{A}_{ij}}$ и расставить их на месте ${{a}_{ij}}$.
  3. Транспонировать эту матрицу $S$, а затем умножить её на некое число $q={1}/{left| A right|};$.

И всё! Обратная матрица ${{A}^{-1}}$ найдена. Давайте посмотрим на примеры:

[left[ begin{matrix} 3 & 1 \ 5 & 2 \end{matrix} right]]

Решение. Проверим обратимость. Посчитаем определитель:

[left| A right|=left| begin{matrix} 3 & 1 \ 5 & 2 \end{matrix} right|=3cdot 2-1cdot 5=6-5=1]

Определитель отличен от нуля. Значит, матрица обратима. Составим союзную матрицу:

Посчитаем алгебраические дополнения:

[begin{align} & {{A}_{11}}={{left(-1 right)}^{1+1}}cdot left| 2 right|=2; \ & {{A}_{12}}={{left(-1 right)}^{1+2}}cdot left| 5 right|=-5; \ & {{A}_{21}}={{left(-1 right)}^{2+1}}cdot left| 1 right|=-1; \ & {{A}_{22}}={{left(-1 right)}^{2+2}}cdot left| 3 right|=3. \ end{align}]

Обратите внимание: определители |2|, |5|, |1| и |3| — это именно определители матриц размера $left[ 1times 1 right]$, а не модули. Т.е. если в определителях стояли отрицательные числа, убирать «минус» не надо.

Итого наша союзная матрица выглядит так:

[{{A}^{-1}}=frac{1}{left| A right|}cdot {{S}^{T}}=frac{1}{1}cdot {{left[ begin{array}{*{35}{r}} 2 & -5 \ -1 & 3 \end{array} right]}^{T}}=left[ begin{array}{*{35}{r}} 2 & -1 \ -5 & 3 \end{array} right]]

Ну вот и всё. Задача решена.

Ответ. $left[ begin{array}{*{35}{r}} 2 & -1 \ -5 & 3 \end{array} right]$

Задача. Найдите обратную матрицу:

[left[ begin{array}{*{35}{r}} 1 & -1 & 2 \ 0 & 2 & -1 \ 1 & 0 & 1 \end{array} right]]

Решение. Опять считаем определитель:

[begin{align} & left| begin{array}{*{35}{r}} 1 & -1 & 2 \ 0 & 2 & -1 \ 1 & 0 & 1 \end{array} right|=begin{matrix} left(1cdot 2cdot 1+left(-1 right)cdot left(-1 right)cdot 1+2cdot 0cdot 0 right)- \ -left(2cdot 2cdot 1+left(-1 right)cdot 0cdot 1+1cdot left(-1 right)cdot 0 right) \end{matrix}= \ & =left(2+1+0 right)-left(4+0+0 right)=-1ne 0. \ end{align}]

Определитель отличен от нуля — матрица обратима. А вот сейчас будет самая жесть: надо посчитать аж 9 (девять, мать их!) алгебраических дополнений. И каждое из них будет содержать определитель $left[ 2times 2 right]$. Полетели:

[begin{matrix} {{A}_{11}}={{left(-1 right)}^{1+1}}cdot left| begin{matrix} 2 & -1 \ 0 & 1 \end{matrix} right|=2; \ {{A}_{12}}={{left(-1 right)}^{1+2}}cdot left| begin{matrix} 0 & -1 \ 1 & 1 \end{matrix} right|=-1; \ {{A}_{13}}={{left(-1 right)}^{1+3}}cdot left| begin{matrix} 0 & 2 \ 1 & 0 \end{matrix} right|=-2; \ … \ {{A}_{33}}={{left(-1 right)}^{3+3}}cdot left| begin{matrix} 1 & -1 \ 0 & 2 \end{matrix} right|=2; \ end{matrix}]

Короче, союзная матрица будет выглядеть так:

Следовательно, обратная матрица будет такой:

[{{A}^{-1}}=frac{1}{-1}cdot left[ begin{matrix} 2 & -1 & -2 \ 1 & -1 & -1 \ -3 & 1 & 2 \end{matrix} right]=left[ begin{array}{*{35}{r}}-2 & -1 & 3 \ 1 & 1 & -1 \ 2 & 1 & -2 \end{array} right]]

Ну и всё. Вот и ответ.

Ответ. $left[ begin{array}{*{35}{r}} -2 & -1 & 3 \ 1 & 1 & -1 \ 2 & 1 & -2 \end{array} right]$

Как видите, в конце каждого примера мы выполняли проверку. В связи с этим важное замечание:

Не ленитесь выполнять проверку. Умножьте исходную матрицу на найденную обратную — должна получиться $E$.

Выполнить эту проверку намного проще и быстрее, чем искать ошибку в дальнейших вычислениях, когда, например, вы решаете матричное уравнение.

Альтернативный способ

Как я и говорил, теорема об обратной матрице прекрасно работает для размеров $left[ 2times 2 right]$ и $left[ 3times 3 right]$ (в последнем случае — уже не так уж и «прекрасно»), а вот для матриц больших размеров начинается прям печаль.

Но не переживайте: есть альтернативный алгоритм, с помощью которого можно невозмутимо найти обратную хоть для матрицы $left[ 10times 10 right]$. Но, как это часто бывает, для рассмотрения этого алгоритма нам потребуется небольшая теоретическая вводная.

Элементарные преобразования

Среди всевозможных преобразований матрицы есть несколько особых — их называют элементарными. Таких преобразований ровно три:

  1. Умножение. Можно взять $i$-ю строку (столбец) и умножить её на любое число $kne 0$;
  2. Сложение. Прибавить к $i$-й строке (столбцу) любую другую $j$-ю строку (столбец), умноженную на любое число $kne 0$ (можно, конечно, и $k=0$, но какой в этом смысл? Ничего не изменится же).
  3. Перестановка. Взять $i$-ю и $j$-ю строки (столбцы) и поменять местами.

Почему эти преобразования называются элементарными (для больших матриц они выглядят не такими уж элементарными) и почему их только три — эти вопросы выходят за рамки сегодняшнего урока. Поэтому не будем вдаваться в подробности.

Важно другое: все эти извращения нам предстоит выполнять над присоединённой матрицей. Да, да: вы не ослышались. Сейчас будет ещё одно определение — последнее в сегодняшнем уроке.

Присоединённая матрица

Наверняка в школе вы решали системы уравнений методом сложения. Ну, там, вычесть из одной строки другую, умножить какую-то строку на число — вот это вот всё.

Так вот: сейчас будет всё то же, но уже «по-взрослому». Готовы?

Определение. Пусть дана матрица $A=left[ ntimes n right]$ и единичная матрица $E$ такого же размера $n$. Тогда присоединённая матрица $left[ Aleft| E right. right]$ — это новая матрица размера $left[ ntimes 2n right]$, которая выглядит так:

[left[ Aleft| E right. right]=left[ begin{array}{rrrr|rrrr}{{a}_{11}} & {{a}_{12}} & … & {{a}_{1n}} & 1 & 0 & … & 0 \{{a}_{21}} & {{a}_{22}} & … & {{a}_{2n}} & 0 & 1 & … & 0 \… & … & … & … & … & … & … & … \{{a}_{n1}} & {{a}_{n2}} & … & {{a}_{nn}} & 0 & 0 & … & 1 \end{array} right]]

Короче говоря, берём матрицу $A$, справа приписываем к ней единичную матрицу $E$ нужного размера, разделяем их вертикальной чертой для красоты — вот вам и присоединённая.:)

В чём прикол? А вот в чём:

Теорема. Пусть матрица $A$ обратима. Рассмотрим присоединённую матрицу $left[ Aleft| E right. right]$. Если с помощью элементарных преобразований строк
привести её к виду $left[ Eleft| B right. right]$, т.е. путём умножения, вычитания и перестановки строк получить из $A$ матрицу $E$ справа, то полученная слева матрица $B$ — это обратная к $A$:

[left[ Aleft| E right. right]to left[ Eleft| B right. right]Rightarrow B={{A}^{-1}}]

Вот так всё просто! Короче говоря, алгоритм нахождения обратной матрицы выглядит так:

  1. Записать присоединённую матрицу $left[ Aleft| E right. right]$;
  2. Выполнять элементарные преобразования строк до тех пор, пока права вместо $A$ не появится $E$;
  3. Разумеется, слева тоже что-то появится — некая матрица $B$. Это и будет обратная;
  4. PROFIT!:)

Конечно, сказать намного проще, чем сделать. Поэтому давайте рассмотрим парочку примеров: для размеров $left[ 3times 3 right]$ и $left[ 4times 4 right]$.

Задача. Найдите обратную матрицу:

[left[ begin{array}{*{35}{r}} 1 & 5 & 1 \ 3 & 2 & 1 \ 6 & -2 & 1 \end{array} right]]

Решение. Составляем присоединённую матрицу:

[left[ begin{array}{rrr|rrr} 1 & 5 & 1 & 1 & 0 & 0 \ 3 & 2 & 1 & 0 & 1 & 0 \ 6 & -2 & 1 & 0 & 0 & 1 \end{array} right]]

Поскольку последний столбец исходной матрицы заполнен единицами, вычтем первую строку из остальных:

[begin{align} & left[ begin{array}{rrr|rrr} 1 & 5 & 1 & 1 & 0 & 0 \ 3 & 2 & 1 & 0 & 1 & 0 \ 6 & -2 & 1 & 0 & 0 & 1 \end{array} right]begin{matrix} downarrow \ -1 \ -1 \end{matrix}to \ & to left[ begin{array}{rrr|rrr} 1 & 5 & 1 & 1 & 0 & 0 \ 2 & -3 & 0 & -1 & 1 & 0 \ 5 & -7 & 0 & -1 & 0 & 1 \end{array} right] \ end{align}]

Больше единиц нет, кроме первой строки. Но её мы не трогаем, иначе в третьем столбце начнут «размножаться» только что убранные единицы.

Зато можем вычесть вторую строку дважды из последней — получим единицу в левом нижнем углу:

[begin{align} & left[ begin{array}{rrr|rrr} 1 & 5 & 1 & 1 & 0 & 0 \ 2 & -3 & 0 & -1 & 1 & 0 \ 5 & -7 & 0 & -1 & 0 & 1 \end{array} right]begin{matrix} \ downarrow \ -2 \end{matrix}to \ & left[ begin{array}{rrr|rrr} 1 & 5 & 1 & 1 & 0 & 0 \ 2 & -3 & 0 & -1 & 1 & 0 \ 1 & -1 & 0 & 1 & -2 & 1 \end{array} right] \ end{align}]

Теперь можно вычесть последнюю строку из первой и дважды из второй — таким образом мы «занулим» первый столбец:

[begin{align} & left[ begin{array}{rrr|rrr} 1 & 5 & 1 & 1 & 0 & 0 \ 2 & -3 & 0 & -1 & 1 & 0 \ 1 & -1 & 0 & 1 & -2 & 1 \end{array} right]begin{matrix} -1 \ -2 \ uparrow \end{matrix}to \ & to left[ begin{array}{rrr|rrr} 0 & 6 & 1 & 0 & 2 & -1 \ 0 & -1 & 0 & -3 & 5 & -2 \ 1 & -1 & 0 & 1 & -2 & 1 \end{array} right] \ end{align}]

Умножим вторую строку на −1, а затем вычтем её 6 раз из первой и прибавим 1 раз к последней:

[begin{align} & left[ begin{array}{rrr|rrr} 0 & 6 & 1 & 0 & 2 & -1 \ 0 & -1 & 0 & -3 & 5 & -2 \ 1 & -1 & 0 & 1 & -2 & 1 \end{array} right]begin{matrix} \ left| cdot left(-1 right) right. \ \end{matrix}to \ & to left[ begin{array}{rrr|rrr} 0 & 6 & 1 & 0 & 2 & -1 \ 0 & 1 & 0 & 3 & -5 & 2 \ 1 & -1 & 0 & 1 & -2 & 1 \end{array} right]begin{matrix} -6 \ updownarrow \ +1 \end{matrix}to \ & to left[ begin{array}{rrr|rrr} 0 & 0 & 1 & -18 & 32 & -13 \ 0 & 1 & 0 & 3 & -5 & 2 \ 1 & 0 & 0 & 4 & -7 & 3 \end{array} right] \ end{align}]

Осталось лишь поменять местами строки 1 и 3:

[left[ begin{array}{rrr|rrr} 1 & 0 & 0 & 4 & -7 & 3 \ 0 & 1 & 0 & 3 & -5 & 2 \ 0 & 0 & 1 & -18 & 32 & -13 \end{array} right]]

Готово! Справа — искомая обратная матрица.

Ответ. $left[ begin{array}{*{35}{r}}4 & -7 & 3 \ 3 & -5 & 2 \ -18 & 32 & -13 \end{array} right]$

Задача. Найдите обратную матрицу:

[left[ begin{matrix} 1 & 4 & 2 & 3 \ 1 & -2 & 1 & -2 \ 1 & -1 & 1 & 1 \ 0 & -10 & -2 & -5 \end{matrix} right]]

Решение. Снова составляем присоединённую:

[left[ begin{array}{rrrr|rrrr} 1 & 4 & 2 & 3 & 1 & 0 & 0 & 0 \ 1 & -2 & 1 & -2 & 0 & 1 & 0 & 0 \ 1 & -1 & 1 & 1 & 0 & 0 & 1 & 0 \ 0 & -10 & -2 & -5 & 0 & 0 & 0 & 1 \end{array} right]]

Немного позалимаем, попечалимся от того, сколько сейчас придётся считать… и начнём считать. Для начала «обнулим» первый столбец, вычитая строку 1 из строк 2 и 3:

[begin{align} & left[ begin{array}{rrrr|rrrr} 1 & 4 & 2 & 3 & 1 & 0 & 0 & 0 \ 1 & -2 & 1 & -2 & 0 & 1 & 0 & 0 \ 1 & -1 & 1 & 1 & 0 & 0 & 1 & 0 \ 0 & -10 & -2 & -5 & 0 & 0 & 0 & 1 \end{array} right]begin{matrix} downarrow \ -1 \ -1 \ \end{matrix}to \ & to left[ begin{array}{rrrr|rrrr} 1 & 4 & 2 & 3 & 1 & 0 & 0 & 0 \ 0 & -6 & -1 & -5 & -1 & 1 & 0 & 0 \ 0 & -5 & -1 & -2 & -1 & 0 & 1 & 0 \ 0 & -10 & -2 & -5 & 0 & 0 & 0 & 1 \end{array} right] \ end{align}]

Наблюдаем слишком много «минусов» в строках 2—4. Умножим все три строки на −1, а затем «выжжем» третий столбец, вычитая строку 3 из остальных:

[begin{align} & left[ begin{array}{rrrr|rrrr} 1 & 4 & 2 & 3 & 1 & 0 & 0 & 0 \ 0 & -6 & -1 & -5 & -1 & 1 & 0 & 0 \ 0 & -5 & -1 & -2 & -1 & 0 & 1 & 0 \ 0 & -10 & -2 & -5 & 0 & 0 & 0 & 1 \end{array} right]begin{matrix} \ left| cdot left(-1 right) right. \ left| cdot left(-1 right) right. \ left| cdot left(-1 right) right. \end{matrix}to \ & to left[ begin{array}{rrrr|rrrr} 1 & 4 & 2 & 3 & 1 & 0 & 0 & 0 \ 0 & 6 & 1 & 5 & 1 & -1 & 0 & 0 \ 0 & 5 & 1 & 2 & 1 & 0 & -1 & 0 \ 0 & 10 & 2 & 5 & 0 & 0 & 0 & -1 \end{array} right]begin{matrix} -2 \ -1 \ updownarrow \ -2 \end{matrix}to \ & to left[ begin{array}{rrrr|rrrr} 1 & -6 & 0 & -1 & -1 & 0 & 2 & 0 \ 0 & 1 & 0 & 3 & 0 & -1 & 1 & 0 \ 0 & 5 & 1 & 2 & 1 & 0 & -1 & 0 \ 0 & 0 & 0 & 1 & -2 & 0 & 2 & -1 \end{array} right] \ end{align}]

Теперь самое время «поджарить» последний столбец исходной матрицы: вычитаем строку 4 из остальных:

[begin{align} & left[ begin{array}{rrrr|rrrr} 1 & -6 & 0 & -1 & -1 & 0 & 2 & 0 \ 0 & 1 & 0 & 3 & 0 & -1 & 1 & 0 \ 0 & 5 & 1 & 2 & 1 & 0 & -1 & 0 \ 0 & 0 & 0 & 1 & -2 & 0 & 2 & -1 \end{array} right]begin{matrix} +1 \ -3 \ -2 \ uparrow \end{matrix}to \ & to left[ begin{array}{rrrr|rrrr} 1 & -6 & 0 & 0 & -3 & 0 & 4 & -1 \ 0 & 1 & 0 & 0 & 6 & -1 & -5 & 3 \ 0 & 5 & 1 & 0 & 5 & 0 & -5 & 2 \ 0 & 0 & 0 & 1 & -2 & 0 & 2 & -1 \end{array} right] \ end{align}]

Финальный бросок: «выжигаем» второй столбец, вычитая строку 2 из строки 1 и 3:

[begin{align} & left[ begin{array}{rrrr|rrrr} 1 & -6 & 0 & 0 & -3 & 0 & 4 & -1 \ 0 & 1 & 0 & 0 & 6 & -1 & -5 & 3 \ 0 & 5 & 1 & 0 & 5 & 0 & -5 & 2 \ 0 & 0 & 0 & 1 & -2 & 0 & 2 & -1 \end{array} right]begin{matrix} 6 \ updownarrow \ -5 \ \end{matrix}to \ & to left[ begin{array}{rrrr|rrrr} 1 & 0 & 0 & 0 & 33 & -6 & -26 & -17 \ 0 & 1 & 0 & 0 & 6 & -1 & -5 & 3 \ 0 & 0 & 1 & 0 & -25 & 5 & 20 & -13 \ 0 & 0 & 0 & 1 & -2 & 0 & 2 & -1 \end{array} right] \ end{align}]

И снова слева единичная матрица, значит справа — обратная.:)

Ответ. $left[ begin{matrix} 33 & -6 & -26 & 17 \ 6 & -1 & -5 & 3 \ -25 & 5 & 20 & -13 \ -2 & 0 & 2 & -1 \end{matrix} right]$

Нахождение обратной матрицы
— задача, которая чаще решается двумя методами:

  • методом алгебраических дополнений, при котором требуется находить определители и транспонировать матрицы;
  • методом исключения неизвестных Гаусса, при котором требуется производить элементарные преобразования матриц
    (складывать строки, умножать строки на одно и то же число и т. д.).

Для особо любознательных существуют и другие методы, например, метод линейных
преобразований. На этом уроке разберём три упомянутых метода и алгоритмы нахождения обратной матрицы
этими методами.

Обратной матрицей

А
, называется такая матрица

А


. (1)

Обратной матрицей

, которую требуется отыскать для данной квадратной матрицы А
, называется такая матрица

произведение на которую матрицы А
справа является единичной матрицей, т.е,

. (1)

Единичной матрицей называется диагональная матрица, у которой все диагональные
элементы равны единице.

Теорема.
Для каждой неособенной (невырожденной, несингулярной) квадратной матрицы можно найти обратную матрицу, и притом только одну. Для особенной (вырожденной, сингулярной) квадратной матрицы обратная матрица не существует.

Квадратная матрица называется неособенной
(или невырожденной
, несингулярной
), если её определитель не равен нулю, и особенной
(или вырожденной
, сингулярной
), если её определитель равен нулю.

Обратная матрица может быть найдена только для квадратной матрицы. Естественно,
обратная матрица также будет квадратной и того же порядка, что и данная матрица. Матрица, для которой
может быть найдена обратная матрица, называется обратимой матрицей.

Для обратной матрицы

существует уместная аналогия с обратным
числом. Для каждого числа a
, не равного нулю, существует такое число b
, что произведение
a
и b
равно единице: ab
= 1
. Число
b
называется обратным для числа b
. Например, для числа 7 обратным является число 1/7,
так как 7*1/7=1.

Нахождение обратной матрицы методом алгебраических дополнений (союзной матрицы)

Для неособенной квадратной матрицы А
обратной является матрица

где —
определитель матрицы А
, а
— матрица, союзная с матрицей А
.

Союзной с квадратной матрицей A
называется матрица
того же порядка, элементами которой являются алгебраические дополнения соответствующих элементов определителя матрицы
,
транспонированной относительно матрицы A. Таким образом, если

то

и

Алгоритм нахождения обратной матрицы методом алгебраических дополнений

1. Найти определитель данной матрицы A
. Если определитель равен нулю, нахождение
обратной матрицы прекращается, так как матрица вырожденная и обратная для неё не существует.

2. Найти матрицу, транспонированную относительно A
.

3. Вычислить элементы союзной матрицы как алгебраические дополнения марицы, найденной на шаге 2.

4. Применить формулу (2): умножить число, обратное определителю матрицы A
,
на союзную матрицу, найденную на шаге 4.

5. Проверить полученный на шаге 4 результат, умножив данную матрицу A
на
обратную матрицу. Если произведение этих матриц равно единичной матрицы, значит обратная матрица была
найдена верно. В противном случае начать процесс решения снова.

Пример 1.
Для матрицы

найти обратную матрицу.

Решение. Для нахождения обратной матрицы необходимо найти определитель матрицы А
.
Находим по правилу треугольников:

Следовательно, матрица А
– неособенная (невырожденная, несингулярная) и для неё существует обратная.

Найдём матрицу, союзную с данной матрицей А
.

Найдём матрицу
,
транспонированную относительно матрицы A
:

Вычисляем элементы союзной матрицы как алгебраические дополнения матрицы,
транспонированной относительно матрицы A
:

Следовательно, матрица
,
союзная с матрицей A
, имеет вид

Замечание.
Порядок вычисления элементов и транспонирования матрицы может
быть иным. Можно сначала вычислить алгебраические дополнения матрицы A
, а затем транспонировать
матрицу алгебраических дополнений. В результате должны получиться те же элементы союзной матрицы.

Применяя формулу (2), находим матрицу, обратную матрице А
:

Нахождение обратной матрицы методом исключения неизвестных Гаусса

Первый шаг для нахождения обратной матрицы методом исключения неизвестных Гаусса —
приписать к матрице A
единичную матрицу того же порядка, отделив их вертикальной чертой. Мы
получим сдвоенную матрицу .
Умножим обе части этой матрицы на ,
тогда получим

,

Алгоритм нахождения обратной матрицы методом исключения неизвестных Гаусса

1. К матрице A
приписать единичную матрицу того же порядка.

2. Полученную сдвоенную матрицу преобразовать так, чтобы в левой её части получилась
единичная матрица, тогда в правой части на месте
единичной матрицы автоматически получится обратная матрица. Матрица A
в левой части
преобразуется в единичную матрицу путём элементарных преобразований матрицы.

2. Если в процессе преобразования матрицы A
в единичную матрицу в какой-либо
строке или в каком-либо столбце окажутся только нули, то определитель матрицы равен
нулю, и, следовательно, матрица A
будет вырожденной, и она не имеет обратной матрицы. В этом
случае дальнейшее нахождение обратной матрицы прекращается.

Пример 2.
Для матрицы

найти обратную матрицу.

и будем её преобразовывать, так чтобы в левой части получилась единичная матрица.
Начинаем преобразования.

Умножим первую строку левой и правой матрицы на (-3) и сложим её со второй строкой,
а затем умножим первую строку на (-4) и сложим её с третьей строкой, тогда получим

.

Чтобы по возможности не было дробных чисел при последующих преобразованиях, создадим
предварительно единицу во второй строке в левой части сдвоенной матрицы. Для этого умножим вторую строку
на 2 и вычтем из неё третью строку, тогда получим

.

Сложим первую строку со второй, а затем умножим вторую строку на (-9) и сложим её
с третьей строкой. Тогда получим

.

Разделим третью строку на 8, тогда

.

Умножим третью строку на 2 и сложим её со второй строкой. Получается:

.

Переставим местами вторую и третью строку, тогда окончательно получим:

.

Видим, что в левой части получилась единичная матрица, следовательно, в правой части
получилась обратная матрица .
Таким образом:

.

Можно проверить правильность вычислений, умножим исходную матрицу на найденную обратную матрицу:

В результате должна получиться обратная матрица.

Пример 3.
Для матрицы

найти обратную матрицу.

Решение. Составляем сдвоенную матрицу

и будем её преобразовывать.

Первую строку умножаем на 3, а вторую на 2, и вычитаем из второй,
а затем первую строку умножаем на 5, а третью на 2 и вычитаем из третьей строки, тогда получим

.

Первую строку умножаем на 2 и складываем её со второй, а затем из третьей строки
вычитаем вторую, тогда получим

.

Видим, что в третьей строке в левой части все элементы получились равными нулю.
Следовательно, матрица вырожденная и обратной матрицы не имеет. Дальнейшее нахождение обратной марицы прекращаем.

Матричная алгебра — Обратная матрица

Обратная матрица

Обратной матрицей
называется матрица, которая при умножении как справа, так и слева на данную матрицу дает единичную матрицу.
Обозначим обратную матрицу к матрице А
через , тогда согласно определению получим:

где Е
– единичная матрица.
Квадратная матрица
называется неособенной
(невырожденной
), если ее определитель не равен нулю. В противном случае она называется особенной
(вырожденной
) или сингулярной
.

Имеет место теорема: всякая неособенная матрица имеет обратную матрицу.

Операция нахождения обратной матрицы называется обращением
матрицы. Рассмотрим алгоритм обращения матрицы. Пусть дана неособенная матрица n
-го порядка:

где Δ = det A
≠ 0.

Алгебраическим дополнением элемента
матрицы n
-го порядка А
называется взятый с определенным знаком определитель матрицы (n
–1)-го порядка, полученной вычеркиванием i
-ой строки и j
-го столбца матрицы А
:

Составим так называемую присоединенную
матрицу:

где– алгебраические дополнения соответствующих элементовматрицы А
.
Заметим, что алгебраические дополнения элементов строк матрицы А
размещаются в соответствующих столбцах матрицы Ã
, то есть одновременно производится транспонирование матрицы.
Разделив все элементы матрицы Ã
на Δ – величину определителя матрицы А
, получим в результате обратную матрицу:

Отметим ряд особых свойств обратной матрицы:
1) для данной матрицы А
ее обратная матрица
является единственной;
2) если существует обратная матрица , то правая обратная
и левая обратная
матрицы совпадают с ней;
3) особенная (вырожденная) квадратная матрица не имеет обратной матрицы.

Основные свойства обратной матрицы:
1) определитель обратной матрицы и определитель исходной матрицы являются обратными величинами;
2) обратная матрица произведения квадратных матриц равна произведениюобратных матриц сомножителей, взятому в обратном порядке:

3) транспонированная обратная матрица равна обратной матрице от данной транспонированной матрицы:

П р и м е р. Вычислить матрицу, обратную данной.

Обратная матрица и её свойства

15 февраля 2018

  • Домашняя работа
  • Ответы

Эта тема является одной из самых ненавистных среди студентов. Хуже, наверное, только определители.

Фишка в том, что само понятие обратного элемента (и я сейчас не только о матрицах) отсылает нас к операции умножения. Даже в школьной программе умножение считается сложной операцией, а уж умножение матриц — вообще отдельная тема, которой у меня посвящён целый параграф и видеоурок.

Сегодня мы не будем вдаваться в подробности матричных вычислений. Просто вспомним: как обозначаются матрицы, как они умножаются и что из этого следует.

Повторение: умножение матриц

Прежде всего договоримся об обозначениях. Матрицей $A$ размера $left[ mtimes n right]$ называется просто таблица из чисел, в которой ровно $m$ строк и $n$ столбцов:

[A=left[ mtimes n right]=underbrace{left[ begin{matrix} {{a}_{11}} & {{a}_{12}} & … & {{a}_{1n}} \ {{a}_{21}} & {{a}_{22}} & … & {{a}_{2n}} \ … & … & … & … \ {{a}_{m1}} & {{a}_{m2}} & … & {{a}_{mn}} \end{matrix} right]}_{n}]

Чтобы случайно не перепутать строки и столбцы местами (поверьте, на экзамене можно и единицу с двойкой перепутать — что уж говорить про какие-то там строки), просто взгляните на картинку:

Определение индексов для клеток матрицы

Что происходит? Если разместить стандартную систему координат $OXY$ в левом верхнем углу и направить оси так, чтобы они охватывали всю матрицу, то каждой клетке этой матрицы можно однозначно сопоставить координаты $left( x;y right)$ — это и будет номер строки и номер столбца.

Почему система координат размещена именно в левом верхнем углу? Да потому что именно оттуда мы начинаем читать любые тексты. Это очень просто запомнить.

А почему ось $x$ направлена именно вниз, а не вправо? Опять всё просто: возьмите стандартную систему координат (ось $x$ идёт вправо, ось $y$ — вверх) и поверните её так, чтобы она охватывала матрицу. Это поворот на 90 градусов по часовой стрелке — его результат мы и видим на картинке.

В общем, как определять индексы у элементов матрицы, мы разобрались. Теперь давайте разберёмся с умножением.

Определение. Матрицы $A=left[ mtimes n right]$ и $B=left[ ntimes k right]$, когда количество столбцов в первой совпадает с количеством строк во второй, называются согласованными.

Именно в таком порядке. Можно сумничать и сказать, мол, матрицы $A$ и $B$ образуют упорядоченную пару $left( A;B right)$: если они согласованы в таком порядке, то совершенно необязательно, что $B$ и $A$, т.е. пара $left( B;A right)$ — тоже согласована.

Умножать можно только согласованные матрицы.

Определение. Произведение согласованных матриц $A=left[ mtimes n right]$ и $B=left[ ntimes k right]$ — это новая матрица $C=left[ mtimes k right]$, элементы которой ${{c}_{ij}}$ считаются по формуле:

[{{c}_{ij}}=sumlimits_{k=1}^{n}{{{a}_{ik}}}cdot {{b}_{kj}}]

Другими словами: чтобы получить элемент ${{c}_{ij}}$ матрицы $C=Acdot B$, нужно взять $i$-строку первой матрицы, $j$-й столбец второй матрицы, а затем попарно перемножить элементы из этой строки и столбца. Результаты сложить.

Да, вот такое суровое определение. Из него сразу следует несколько фактов:

  1. Умножение матриц, вообще говоря, некоммутативно: $Acdot Bne Bcdot A$;
  2. Однако умножение ассоциативно: $left( Acdot B right)cdot C=Acdot left( Bcdot C right)$;
  3. И даже дистрибутивно: $left( A+B right)cdot C=Acdot C+Bcdot C$;
  4. И ещё раз дистрибутивно: $Acdot left( B+C right)=Acdot B+Acdot C$.

Дистрибутивность умножения пришлось отдельно описывать для левого и правого множителя-суммы как раз из-за некоммутативности операции умножения.

Если всё же получается так, что $Acdot B=Bcdot A$, такие матрицы называются перестановочными.

Среди всех матриц, которые там на что-то умножаются, есть особые — те, которые при умножении на любую матрицу $A$ снова дают $A$:

Определение. Матрица $E$ называется единичной, если $Acdot E=A$ или $Ecdot A=A$. В случае с квадратной матрицей $A$ можем записать:

[Acdot E=Ecdot A=A]

Единичная матрица — частый гость при решении матричных уравнений. И вообще частый гость в мире матриц.:)

А ещё из-за этой $E$ кое-кто придумал всю ту дичь, которая будет написана дальше.

Что такое обратная матрица

Поскольку умножение матриц — весьма трудоёмкая операция (приходится перемножать кучу строчек и столбцов), то понятие обратной матрицы тоже оказывается не самым тривиальным. И требующим некоторых пояснений.

Ключевое определение

Что ж, пора познать истину.

Определение. Матрица $B$ называется обратной к матрице $A$, если

[Acdot B=Bcdot A=E]

Обратная матрица обозначается через ${{A}^{-1}}$ (не путать со степенью!), поэтому определение можно переписать так:

[Acdot {{A}^{-1}}={{A}^{-1}}cdot A=E]

Казалось бы, всё предельно просто и ясно. Но при анализе такого определения сразу возникает несколько вопросов:

  1. Всегда ли существует обратная матрица? И если не всегда, то как определить: когда она существует, а когда — нет?
  2. А кто сказал, что такая матрица ровно одна? Вдруг для некоторой исходной матрицы $A$ найдётся целая толпа обратных?
  3. Как выглядят все эти «обратные»? И как, собственно, их считать?

Насчёт алгоритмов вычисления — об этом мы поговорим чуть позже. Но на остальные вопросы ответим прямо сейчас. Оформим их в виде отдельных утверждений-лемм.

Основные свойства

Начнём с того, как в принципе должна выглядеть матрица $A$, чтобы для неё существовала ${{A}^{-1}}$. Сейчас мы убедимся в том, что обе эти матрицы должны быть квадратными, причём одного размера: $left[ ntimes n right]$.

Лемма 1. Дана матрица $A$ и обратная ей ${{A}^{-1}}$. Тогда обе эти матрицы — квадратные, причём одинакового порядка $n$.

Доказательство. Всё просто. Пусть матрица $A=left[ mtimes n right]$, ${{A}^{-1}}=left[ atimes b right]$. Поскольку произведение $Acdot {{A}^{-1}}=E$ по определению существует, матрицы $A$ и ${{A}^{-1}}$ согласованы в указанном порядке:

[begin{align} & left[ mtimes n right]cdot left[ atimes b right]=left[ mtimes b right] \ & n=a end{align}]

Это прямое следствие из алгоритма перемножения матриц: коэффициенты $n$ и $a$ являются «транзитными» и должны быть равны.

Вместе с тем определено и обратное умножение: ${{A}^{-1}}cdot A=E$, поэтому матрицы ${{A}^{-1}}$ и $A$ тоже согласованы в указанном порядке:

[begin{align} & left[ atimes b right]cdot left[ mtimes n right]=left[ atimes n right] \ & b=m end{align}]

Таким образом, без ограничения общности можем считать, что $A=left[ mtimes n right]$, ${{A}^{-1}}=left[ ntimes m right]$. Однако согласно определению $Acdot {{A}^{-1}}={{A}^{-1}}cdot A$, поэтому размеры матриц строго совпадают:

[begin{align} & left[ mtimes n right]=left[ ntimes m right] \ & m=n end{align}]

Вот и получается, что все три матрицы — $A$, ${{A}^{-1}}$ и $E$ — являются квадратными размером $left[ ntimes n right]$. Лемма доказана.

Что ж, уже неплохо. Мы видим, что обратимыми бывают лишь квадратные матрицы. Теперь давайте убедимся, что обратная матрица всегда одна.

Лемма 2. Дана матрица $A$ и обратная ей ${{A}^{-1}}$. Тогда эта обратная матрица — единственная.

Доказательство. Пойдём от противного: пусть у матрицы $A$ есть хотя бы два экземпляра обратных —$B$ и $C$. Тогда, согласно определению, верны следующие равенства:

[begin{align} & Acdot B=Bcdot A=E; \ & Acdot C=Ccdot A=E. \ end{align}]

Из леммы 1 мы заключаем, что все четыре матрицы — $A$, $B$, $C$ и $E$ — являются квадратными одинакового порядка: $left[ ntimes n right]$. Следовательно, определено произведение:

[Bcdot Acdot C]

Поскольку умножение матриц ассоциативно (но не коммутативно!), мы можем записать:

[begin{align} & Bcdot Acdot C=left( Bcdot A right)cdot C=Ecdot C=C; \ & Bcdot Acdot C=Bcdot left( Acdot C right)=Bcdot E=B; \ & Bcdot Acdot C=C=BRightarrow B=C. \ end{align}]

Получили единственно возможный вариант: два экземпляра обратной матрицы равны. Лемма доказана.

Приведённые рассуждения почти дословно повторяют доказательство единственность обратного элемента для всех действительных чисел $bne 0$. Единственное существенное дополнение — учёт размерности матриц.

Впрочем, мы до сих пор ничего не знаем о том, всякая ли квадратная матрица является обратимой. Тут нам на помощь приходит определитель — это ключевая характеристика для всех квадратных матриц.

Лемма 3. Дана матрица $A$. Если обратная к ней матрица ${{A}^{-1}}$ существует, то определитель исходной матрицы отличен от нуля:

[left| A right|ne 0]

Доказательство. Мы уже знаем, что $A$ и ${{A}^{-1}}$ — квадратные матрицы размера $left[ ntimes n right]$. Следовательно, для каждой из них можно вычислить определитель: $left| A right|$ и $left| {{A}^{-1}} right|$. Однако определитель произведения равен произведению определителей:

[left| Acdot B right|=left| A right|cdot left| B right|Rightarrow left| Acdot {{A}^{-1}} right|=left| A right|cdot left| {{A}^{-1}} right|]

Но согласно определению $Acdot {{A}^{-1}}=E$, а определитель $E$ всегда равен 1, поэтому

[begin{align} & Acdot {{A}^{-1}}=E; \ & left| Acdot {{A}^{-1}} right|=left| E right|; \ & left| A right|cdot left| {{A}^{-1}} right|=1. \ end{align}]

Произведение двух чисел равно единице только в том случае, когда каждое из этих чисел отлично от нуля:

[left| A right|ne 0;quad left| {{A}^{-1}} right|ne 0.]

Вот и получается, что $left| A right|ne 0$. Лемма доказана.

На самом деле это требование вполне логично. Сейчас мы разберём алгоритм нахождения обратной матрицы — и станет совершенно ясно, почему при нулевом определителе никакой обратной матрицы в принципе не может существовать.

Но для начала сформулируем «вспомогательное» определение:

Определение. Вырожденная матрица — это квадратная матрица размера $left[ ntimes n right]$, чей определитель равен нулю.

Таким образом, мы можем утверждать, что всякая обратимая матрица является невырожденной.

Как найти обратную матрицу

Сейчас мы рассмотрим универсальный алгоритм нахождения обратных матриц. Вообще, существует два общепринятых алгоритма, и второй мы тоже сегодня рассмотрим.

Тот, который будет рассмотрен сейчас, очень эффективен для матриц размера $left[ 2times 2 right]$ и — частично — размера $left[ 3times 3 right]$. А вот начиная с размера $left[ 4times 4 right]$ его лучше не применять. Почему — сейчас сами всё поймёте.

Алгебраические дополнения

Готовьтесь. Сейчас будет боль. Нет, не переживайте: к вам не идёт красивая медсестра в юбке, чулках с кружевами и не сделает укол в ягодицу. Всё куда прозаичнее: к вам идут алгебраические дополнения и Её Величество «Союзная Матрица».

Начнём с главного. Пусть имеется квадратная матрица размера $A=left[ ntimes n right]$, элементы которой именуются ${{a}_{ij}}$. Тогда для каждого такого элемента можно определить алгебраическое дополнение:

Определение. Алгебраическое дополнение ${{A}_{ij}}$ к элементу ${{a}_{ij}}$, стоящего в $i$-й строке и $j$-м столбце матрицы $A=left[ ntimes n right]$ — это конструкция вида

[{{A}_{ij}}={{left( -1 right)}^{i+j}}cdot M_{ij}^{*}]

Где $M_{ij}^{*}$ — определитель матрицы, полученной из исходной $A$ вычёркиванием той самой $i$-й строки и $j$-го столбца.

Ещё раз. Алгебраическое дополнение к элементу матрицы с координатами $left( i;j right)$ обозначается как ${{A}_{ij}}$ и считается по схеме:

  1. Сначала вычёркиваем из исходной матрицы $i$-строчку и $j$-й столбец. Получим новую квадратную матрицу, и её определитель мы обозначаем как $M_{ij}^{*}$.
  2. Затем умножаем этот определитель на ${{left( -1 right)}^{i+j}}$ — поначалу это выражение может показаться мозговыносящим, но по сути мы просто выясняем знак перед $M_{ij}^{*}$.
  3. Считаем — получаем конкретное число. Т.е. алгебраическое дополнение — это именно число, а не какая-то новая матрица и т.д.

Саму матрицу $M_{ij}^{*}$ называют дополнительным минором к элементу ${{a}_{ij}}$. И в этом смысле приведённое выше определение алгебраического дополнения является частным случаем более сложного определения — того, что мы рассматривали в уроке про определитель.

Важное замечание. Вообще-то во «взрослой» математике алгебраические дополнения определяются так:

  1. Берём в квадратной матрице $k$ строчек и $k$ столбцов. На их пересечении получится матрица размера $left[ ktimes k right]$ — её определитель называется минором порядка $k$ и обозначается ${{M}_{k}}$.
  2. Затем вычёркиваем эти «избранные» $k$ строчек и $k$ столбцов. Снова получится квадратная матрица — её определитель называется дополнительным минором и обозначается $M_{k}^{*}$.
  3. Умножаем $M_{k}^{*}$ на ${{left( -1 right)}^{t}}$, где $t$ — это (вот сейчас внимание!) сумма номеров всех выбранных строчек и столбцов. Это и будет алгебраическое дополнение.

Взгляните на третий шаг: там вообще-то сумма $2k$ слагаемых! Другое дело, что для $k=1$ мы получим лишь 2 слагаемых — это и будут те самые $i+j$ — «координаты» элемента ${{a}_{ij}}$, для которого мы ищем алгебраическое дополнение.

Таким образом сегодня мы используем слегка упрощённое определение. Но как мы увидим в дальнейшем, его окажется более чем достаточно. Куда важнее следующая штука:

Определение. Союзная матрица $S$ к квадратной матрице $A=left[ ntimes n right]$ — это новая матрица размера $left[ ntimes n right]$, которая получается из $A$ заменой ${{a}_{ij}}$ алгебраическими дополнениями ${{A}_{ij}}$:

[A=left[ begin{matrix} {{a}_{11}} & {{a}_{12}} & … & {{a}_{1n}} \ {{a}_{21}} & {{a}_{22}} & … & {{a}_{2n}} \ … & … & … & … \ {{a}_{n1}} & {{a}_{n2}} & … & {{a}_{nn}} \end{matrix} right]Rightarrow S=left[ begin{matrix} {{A}_{11}} & {{A}_{12}} & … & {{A}_{1n}} \ {{A}_{21}} & {{A}_{22}} & … & {{A}_{2n}} \ … & … & … & … \ {{A}_{n1}} & {{A}_{n2}} & … & {{A}_{nn}} \end{matrix} right]]

Первая мысль, возникающая в момент осознания этого определения — «это сколько же придётся всего считать!» Расслабьтесь: считать придётся, но не так уж и много.:)

Что ж, всё это очень мило, но зачем это нужно? А вот зачем.

Основная теорема

Вернёмся немного назад. Помните, в Лемме 3 утверждалось, что обратимая матрица $A$ всегда не вырождена (т.е. её определитель отличен от нуля: $left| A right|ne 0$).

Так вот, верно и обратное: если матрица $A$ не вырождена, то она всегда обратима. И даже существует схема поиска ${{A}^{-1}}$. Зацените:

Теорема об обратной матрице. Пусть дана квадратная матрица $A=left[ ntimes n right]$, причём её определитель отличен от нуля: $left| A right|ne 0$. Тогда обратная матрица ${{A}^{-1}}$ существует и считается по формуле:

[{{A}^{-1}}=frac{1}{left| A right|}cdot {{S}^{T}}]

А теперь — всё то же самое, но разборчивым почерком. Чтобы найти обратную матрицу, нужно:

  1. Посчитать определитель $left| A right|$ и убедиться, что он отличен от нуля.
  2. Составить союзную матрицу $S$, т.е. посчитать 100500 алгебраических дополнений ${{A}_{ij}}$ и расставить их на месте ${{a}_{ij}}$.
  3. Транспонировать эту матрицу $S$, а затем умножить её на некое число $q={1}/{left| A right|};$.

И всё! Обратная матрица ${{A}^{-1}}$ найдена. Давайте посмотрим на примеры:

Задача. Найдите обратную матрицу:

[left[ begin{matrix} 3 & 1 \ 5 & 2 \end{matrix} right]]

Решение. Проверим обратимость. Посчитаем определитель:

[left| A right|=left| begin{matrix} 3 & 1 \ 5 & 2 \end{matrix} right|=3cdot 2-1cdot 5=6-5=1]

Определитель отличен от нуля. Значит, матрица обратима. Составим союзную матрицу:

[S=left[ begin{matrix} {{A}_{11}} & {{A}_{12}} \ {{A}_{21}} & {{A}_{22}} \end{matrix} right]]

Посчитаем алгебраические дополнения:

[begin{align} & {{A}_{11}}={{left( -1 right)}^{1+1}}cdot left| 2 right|=2; \ & {{A}_{12}}={{left( -1 right)}^{1+2}}cdot left| 5 right|=-5; \ & {{A}_{21}}={{left( -1 right)}^{2+1}}cdot left| 1 right|=-1; \ & {{A}_{22}}={{left( -1 right)}^{2+2}}cdot left| 3 right|=3. \ end{align}]

Обратите внимание: определители |2|, |5|, |1| и |3| — это именно определители матриц размера $left[ 1times 1 right]$, а не модули. Т.е. если в определителях стояли отрицательные числа, убирать «минус» не надо.

Итого наша союзная матрица выглядит так:

[S=left[ begin{array}{*{35}{r}}2 & -5 \ -1 & 3 \end{array} right]]

Осталось посчитать обратную:

[{{A}^{-1}}=frac{1}{left| A right|}cdot {{S}^{T}}=frac{1}{1}cdot {{left[ begin{array}{*{35}{r}} 2 & -5 \ -1 & 3 \end{array} right]}^{T}}=left[ begin{array}{*{35}{r}} 2 & -1 \ -5 & 3 \end{array} right]]

Ну вот и всё. Задача решена.

Ответ. $left[ begin{array}{*{35}{r}} 2 & -1 \ -5 & 3 \end{array} right]$

Задача. Найдите обратную матрицу:

[left[ begin{array}{*{35}{r}} 1 & -1 & 2 \ 0 & 2 & -1 \ 1 & 0 & 1 \end{array} right]]

Решение. Опять считаем определитель:

[begin{align} & left| begin{array}{*{35}{r}} 1 & -1 & 2 \ 0 & 2 & -1 \ 1 & 0 & 1 \end{array} right|=begin{matrix} left( 1cdot 2cdot 1+left( -1 right)cdot left( -1 right)cdot 1+2cdot 0cdot 0 right)- \ -left( 2cdot 2cdot 1+left( -1 right)cdot 0cdot 1+1cdot left( -1 right)cdot 0 right) \end{matrix}= \ & =left( 2+1+0 right)-left( 4+0+0 right)=-1ne 0. \ end{align}]

Определитель отличен от нуля — матрица обратима. А вот сейчас будет самая жесть: надо посчитать аж 9 (девять, мать их!) алгебраических дополнений. И каждое из них будет содержать определитель $left[ 2times 2 right]$. Полетели:

[begin{matrix} {{A}_{11}}={{left( -1 right)}^{1+1}}cdot left| begin{matrix} 2 & -1 \ 0 & 1 \end{matrix} right|=2; \ {{A}_{12}}={{left( -1 right)}^{1+2}}cdot left| begin{matrix} 0 & -1 \ 1 & 1 \end{matrix} right|=-1; \ {{A}_{13}}={{left( -1 right)}^{1+3}}cdot left| begin{matrix} 0 & 2 \ 1 & 0 \end{matrix} right|=-2; \ … \ {{A}_{33}}={{left( -1 right)}^{3+3}}cdot left| begin{matrix} 1 & -1 \ 0 & 2 \end{matrix} right|=2; \ end{matrix}]

Короче, союзная матрица будет выглядеть так:

[S=left[ begin{matrix} 2 & -1 & -2 \ 1 & -1 & -1 \ -3 & 1 & 2 \end{matrix} right]]

Следовательно, обратная матрица будет такой:

[{{A}^{-1}}=frac{1}{-1}cdot left[ begin{matrix} 2 & -1 & -2 \ 1 & -1 & -1 \ -3 & 1 & 2 \end{matrix} right]=left[ begin{array}{*{35}{r}}-2 & -1 & 3 \ 1 & 1 & -1 \ 2 & 1 & -2 \end{array} right]]

Ну и всё. Вот и ответ.

Ответ. $left[ begin{array}{*{35}{r}} -2 & -1 & 3 \ 1 & 1 & -1 \ 2 & 1 & -2 \end{array} right]$

Как видите, в конце каждого примера мы выполняли проверку. В связи с этим важное замечание:

Не ленитесь выполнять проверку. Умножьте исходную матрицу на найденную обратную — должна получиться $E$.

Выполнить эту проверку намного проще и быстрее, чем искать ошибку в дальнейших вычислениях, когда, например, вы решаете матричное уравнение.

Альтернативный способ

Как я и говорил, теорема об обратной матрице прекрасно работает для размеров $left[ 2times 2 right]$ и $left[ 3times 3 right]$ (в последнем случае — уже не так уж и «прекрасно»), а вот для матриц больших размеров начинается прям печаль.

Но не переживайте: есть альтернативный алгоритм, с помощью которого можно невозмутимо найти обратную хоть для матрицы $left[ 10times 10 right]$. Но, как это часто бывает, для рассмотрения этого алгоритма нам потребуется небольшая теоретическая вводная.

Элементарные преобразования

Среди всевозможных преобразований матрицы есть несколько особых — их называют элементарными. Таких преобразований ровно три:

  1. Умножение. Можно взять $i$-ю строку (столбец) и умножить её на любое число $kne 0$;
  2. Сложение. Прибавить к $i$-й строке (столбцу) любую другую $j$-ю строку (столбец), умноженную на любое число $kne 0$ (можно, конечно, и $k=0$, но какой в этом смысл? Ничего не изменится же).
  3. Перестановка. Взять $i$-ю и $j$-ю строки (столбцы) и поменять местами.

Почему эти преобразования называются элементарными (для больших матриц они выглядят не такими уж элементарными) и почему их только три — эти вопросы выходят за рамки сегодняшнего урока. Поэтому не будем вдаваться в подробности.

Важно другое: все эти извращения нам предстоит выполнять над присоединённой матрицей. Да, да: вы не ослышались. Сейчас будет ещё одно определение — последнее в сегодняшнем уроке.

Присоединённая матрица

Наверняка в школе вы решали системы уравнений методом сложения. Ну, там, вычесть из одной строки другую, умножить какую-то строку на число — вот это вот всё.

Так вот: сейчас будет всё то же, но уже «по-взрослому». Готовы?

Определение. Пусть дана матрица $A=left[ ntimes n right]$ и единичная матрица $E$ такого же размера $n$. Тогда присоединённая матрица $left[ Aleft| E right. right]$ — это новая матрица размера $left[ ntimes 2n right]$, которая выглядит так:

[left[ Aleft| E right. right]=left[ begin{array}{rrrr|rrrr}{{a}_{11}} & {{a}_{12}} & … & {{a}_{1n}} & 1 & 0 & … & 0 \{{a}_{21}} & {{a}_{22}} & … & {{a}_{2n}} & 0 & 1 & … & 0 \… & … & … & … & … & … & … & … \{{a}_{n1}} & {{a}_{n2}} & … & {{a}_{nn}} & 0 & 0 & … & 1 \end{array} right]]

Короче говоря, берём матрицу $A$, справа приписываем к ней единичную матрицу $E$ нужного размера, разделяем их вертикальной чертой для красоты — вот вам и присоединённая.:)

В чём прикол? А вот в чём:

Теорема. Пусть матрица $A$ обратима. Рассмотрим присоединённую матрицу $left[ Aleft| E right. right]$. Если с помощью элементарных преобразований строк привести её к виду $left[ Eleft| B right. right]$, т.е. путём умножения, вычитания и перестановки строк получить из $A$ матрицу $E$ справа, то полученная слева матрица $B$ — это обратная к $A$:

[left[ Aleft| E right. right]to left[ Eleft| B right. right]Rightarrow B={{A}^{-1}}]

Вот так всё просто! Короче говоря, алгоритм нахождения обратной матрицы выглядит так:

  1. Записать присоединённую матрицу $left[ Aleft| E right. right]$;
  2. Выполнять элементарные преобразования строк до тех пор, пока права вместо $A$ не появится $E$;
  3. Разумеется, слева тоже что-то появится — некая матрица $B$. Это и будет обратная;
  4. PROFIT!:)

Конечно, сказать намного проще, чем сделать. Поэтому давайте рассмотрим парочку примеров: для размеров $left[ 3times 3 right]$ и $left[ 4times 4 right]$.

Задача. Найдите обратную матрицу:

[left[ begin{array}{*{35}{r}} 1 & 5 & 1 \ 3 & 2 & 1 \ 6 & -2 & 1 \end{array} right]]

Решение. Составляем присоединённую матрицу:

[left[ begin{array}{rrr|rrr} 1 & 5 & 1 & 1 & 0 & 0 \ 3 & 2 & 1 & 0 & 1 & 0 \ 6 & -2 & 1 & 0 & 0 & 1 \end{array} right]]

Поскольку последний столбец исходной матрицы заполнен единицами, вычтем первую строку из остальных:

[begin{align} & left[ begin{array}{rrr|rrr} 1 & 5 & 1 & 1 & 0 & 0 \ 3 & 2 & 1 & 0 & 1 & 0 \ 6 & -2 & 1 & 0 & 0 & 1 \end{array} right]begin{matrix} downarrow \ -1 \ -1 \end{matrix}to \ & to left[ begin{array}{rrr|rrr} 1 & 5 & 1 & 1 & 0 & 0 \ 2 & -3 & 0 & -1 & 1 & 0 \ 5 & -7 & 0 & -1 & 0 & 1 \end{array} right] \ end{align}]

Больше единиц нет, кроме первой строки. Но её мы не трогаем, иначе в третьем столбце начнут «размножаться» только что убранные единицы.

Зато можем вычесть вторую строку дважды из последней — получим единицу в левом нижнем углу:

[begin{align} & left[ begin{array}{rrr|rrr} 1 & 5 & 1 & 1 & 0 & 0 \ 2 & -3 & 0 & -1 & 1 & 0 \ 5 & -7 & 0 & -1 & 0 & 1 \end{array} right]begin{matrix} \ downarrow \ -2 \end{matrix}to \ & left[ begin{array}{rrr|rrr} 1 & 5 & 1 & 1 & 0 & 0 \ 2 & -3 & 0 & -1 & 1 & 0 \ 1 & -1 & 0 & 1 & -2 & 1 \end{array} right] \ end{align}]

Теперь можно вычесть последнюю строку из первой и дважды из второй — таким образом мы «занулим» первый столбец:

[begin{align} & left[ begin{array}{rrr|rrr} 1 & 5 & 1 & 1 & 0 & 0 \ 2 & -3 & 0 & -1 & 1 & 0 \ 1 & -1 & 0 & 1 & -2 & 1 \end{array} right]begin{matrix} -1 \ -2 \ uparrow \end{matrix}to \ & to left[ begin{array}{rrr|rrr} 0 & 6 & 1 & 0 & 2 & -1 \ 0 & -1 & 0 & -3 & 5 & -2 \ 1 & -1 & 0 & 1 & -2 & 1 \end{array} right] \ end{align}]

Умножим вторую строку на −1, а затем вычтем её 6 раз из первой и прибавим 1 раз к последней:

[begin{align} & left[ begin{array}{rrr|rrr} 0 & 6 & 1 & 0 & 2 & -1 \ 0 & -1 & 0 & -3 & 5 & -2 \ 1 & -1 & 0 & 1 & -2 & 1 \end{array} right]begin{matrix} \ left| cdot left( -1 right) right. \ \end{matrix}to \ & to left[ begin{array}{rrr|rrr} 0 & 6 & 1 & 0 & 2 & -1 \ 0 & 1 & 0 & 3 & -5 & 2 \ 1 & -1 & 0 & 1 & -2 & 1 \end{array} right]begin{matrix} -6 \ updownarrow \ +1 \end{matrix}to \ & to left[ begin{array}{rrr|rrr} 0 & 0 & 1 & -18 & 32 & -13 \ 0 & 1 & 0 & 3 & -5 & 2 \ 1 & 0 & 0 & 4 & -7 & 3 \end{array} right] \ end{align}]

Осталось лишь поменять местами строки 1 и 3:

[left[ begin{array}{rrr|rrr} 1 & 0 & 0 & 4 & -7 & 3 \ 0 & 1 & 0 & 3 & -5 & 2 \ 0 & 0 & 1 & -18 & 32 & -13 \end{array} right]]

Готово! Справа — искомая обратная матрица.

Ответ. $left[ begin{array}{*{35}{r}}4 & -7 & 3 \ 3 & -5 & 2 \ -18 & 32 & -13 \end{array} right]$

Задача. Найдите обратную матрицу:

[left[ begin{matrix} 1 & 4 & 2 & 3 \ 1 & -2 & 1 & -2 \ 1 & -1 & 1 & 1 \ 0 & -10 & -2 & -5 \end{matrix} right]]

Решение. Снова составляем присоединённую:

[left[ begin{array}{rrrr|rrrr} 1 & 4 & 2 & 3 & 1 & 0 & 0 & 0 \ 1 & -2 & 1 & -2 & 0 & 1 & 0 & 0 \ 1 & -1 & 1 & 1 & 0 & 0 & 1 & 0 \ 0 & -10 & -2 & -5 & 0 & 0 & 0 & 1 \end{array} right]]

Немного позалимаем, попечалимся от того, сколько сейчас придётся считать… и начнём считать. Для начала «обнулим» первый столбец, вычитая строку 1 из строк 2 и 3:

[begin{align} & left[ begin{array}{rrrr|rrrr} 1 & 4 & 2 & 3 & 1 & 0 & 0 & 0 \ 1 & -2 & 1 & -2 & 0 & 1 & 0 & 0 \ 1 & -1 & 1 & 1 & 0 & 0 & 1 & 0 \ 0 & -10 & -2 & -5 & 0 & 0 & 0 & 1 \end{array} right]begin{matrix} downarrow \ -1 \ -1 \ \end{matrix}to \ & to left[ begin{array}{rrrr|rrrr} 1 & 4 & 2 & 3 & 1 & 0 & 0 & 0 \ 0 & -6 & -1 & -5 & -1 & 1 & 0 & 0 \ 0 & -5 & -1 & -2 & -1 & 0 & 1 & 0 \ 0 & -10 & -2 & -5 & 0 & 0 & 0 & 1 \end{array} right] \ end{align}]

Наблюдаем слишком много «минусов» в строках 2—4. Умножим все три строки на −1, а затем «выжжем» третий столбец, вычитая строку 3 из остальных:

[begin{align} & left[ begin{array}{rrrr|rrrr} 1 & 4 & 2 & 3 & 1 & 0 & 0 & 0 \ 0 & -6 & -1 & -5 & -1 & 1 & 0 & 0 \ 0 & -5 & -1 & -2 & -1 & 0 & 1 & 0 \ 0 & -10 & -2 & -5 & 0 & 0 & 0 & 1 \end{array} right]begin{matrix} \ left| cdot left( -1 right) right. \ left| cdot left( -1 right) right. \ left| cdot left( -1 right) right. \end{matrix}to \ & to left[ begin{array}{rrrr|rrrr} 1 & 4 & 2 & 3 & 1 & 0 & 0 & 0 \ 0 & 6 & 1 & 5 & 1 & -1 & 0 & 0 \ 0 & 5 & 1 & 2 & 1 & 0 & -1 & 0 \ 0 & 10 & 2 & 5 & 0 & 0 & 0 & -1 \end{array} right]begin{matrix} -2 \ -1 \ updownarrow \ -2 \end{matrix}to \ & to left[ begin{array}{rrrr|rrrr} 1 & -6 & 0 & -1 & -1 & 0 & 2 & 0 \ 0 & 1 & 0 & 3 & 0 & -1 & 1 & 0 \ 0 & 5 & 1 & 2 & 1 & 0 & -1 & 0 \ 0 & 0 & 0 & 1 & -2 & 0 & 2 & -1 \end{array} right] \ end{align}]

Теперь самое время «поджарить» последний столбец исходной матрицы: вычитаем строку 4 из остальных:

[begin{align} & left[ begin{array}{rrrr|rrrr} 1 & -6 & 0 & -1 & -1 & 0 & 2 & 0 \ 0 & 1 & 0 & 3 & 0 & -1 & 1 & 0 \ 0 & 5 & 1 & 2 & 1 & 0 & -1 & 0 \ 0 & 0 & 0 & 1 & -2 & 0 & 2 & -1 \end{array} right]begin{matrix} +1 \ -3 \ -2 \ uparrow \end{matrix}to \ & to left[ begin{array}{rrrr|rrrr} 1 & -6 & 0 & 0 & -3 & 0 & 4 & -1 \ 0 & 1 & 0 & 0 & 6 & -1 & -5 & 3 \ 0 & 5 & 1 & 0 & 5 & 0 & -5 & 2 \ 0 & 0 & 0 & 1 & -2 & 0 & 2 & -1 \end{array} right] \ end{align}]

Финальный бросок: «выжигаем» второй столбец, вычитая строку 2 из строки 1 и 3:

[begin{align} & left[ begin{array}{rrrr|rrrr} 1 & -6 & 0 & 0 & -3 & 0 & 4 & -1 \ 0 & 1 & 0 & 0 & 6 & -1 & -5 & 3 \ 0 & 5 & 1 & 0 & 5 & 0 & -5 & 2 \ 0 & 0 & 0 & 1 & -2 & 0 & 2 & -1 \end{array} right]begin{matrix} 6 \ updownarrow \ -5 \ \end{matrix}to \ & to left[ begin{array}{rrrr|rrrr} 1 & 0 & 0 & 0 & 33 & -6 & -26 & -17 \ 0 & 1 & 0 & 0 & 6 & -1 & -5 & 3 \ 0 & 0 & 1 & 0 & -25 & 5 & 20 & -13 \ 0 & 0 & 0 & 1 & -2 & 0 & 2 & -1 \end{array} right] \ end{align}]

И снова слева единичная матрица, значит справа — обратная.:)

Ответ. $left[ begin{matrix} 33 & -6 & -26 & 17 \ 6 & -1 & -5 & 3 \ -25 & 5 & 20 & -13 \ -2 & 0 & 2 & -1 \end{matrix} right]$

Ну вот и всё. Проверку сделайте сами — мне в лом.:)

Смотрите также:

  1. Определитель
  2. Дополнительные соображения
  3. Тест к уроку «Площади многоугольников на координатной сетке» (легкий)
  4. Четырехугольная пирамида в задаче C2
  5. Задача B5: площадь кольца
  6. Случай четырехугольной пирамиды

Решение матричных уравнений

Финальная глава саги.

Линейная алгебра и, в частности, матрицы — это основа математики нейросетей. Когда говорят «машинное обучение», на самом деле говорят «перемножение матриц», «решение матричных уравнений» и «поиск коэффициентов в матричных уравнениях».

Понятно, что между простой матрицей в линейной алгебре и нейросетью, которая генерирует котов, много слоёв усложнений, дополнительной логики, обучения и т. д. Но здесь мы говорим именно о фундаменте. Цель — чтобы стало понятно, из чего оно сделано.

Краткое содержание прошлых частей:

  • Линейная алгебра изучает векторы, матрицы и другие понятия, которые относятся к упорядоченным наборам данных. Линейной алгебре интересно, как можно трансформировать эти упорядоченные данные, складывать и умножать, всячески обсчитывать и находить в них закономерности.
  • Вектор — это набор упорядоченных данных в одном измерении. Можно упрощённо сказать, что это последовательность чисел.
  • Матрица — это тоже набор упорядоченных данных, только уже не в одном измерении, а в двух (или даже больше).
  • Матрицу можно представить как упорядоченную сумку с данными. И с этой сумкой как с единым целым можно совершать какие-то действия. Например, делить, умножать, менять знаки.
  • Матрицы можно складывать и умножать на другие матрицы. Это как взять две сумки с данными и получить третью сумку, тоже с данными, только теперь какими-то новыми.
  • Матрицы перемножаются по довольно замороченному алгоритму. Арифметика простая, а порядок перемножения довольно запутанный.

И вот наконец мы здесь: если мы можем перемножать матрицы, то мы можем и решить матричное уравнение.

❌ Никакого практического применения следующего материала в народном хозяйстве вы не увидите. Это чистая алгебра в несколько упрощённом виде. Отсюда до практики далёкий путь, поэтому, если нужно что-то практическое, — посмотрите, как мы генерим Чехова на цепях Маркова.

Что такое матричное уравнение

Матричное уравнение — это когда мы умножаем известную матрицу на матрицу Х и получаем новую матрицу. Наша задача — найти неизвестную матрицу Х.

Шаг 1. Упрощаем уравнение

Вместо известных числовых матриц вводим в уравнение буквы: первую матрицу обозначаем буквой A, вторую — буквой B. Неизвестную матрицу X оставляем. Это упрощение поможет составить формулу и выразить X через известную матрицу.

Приводим матричное уравнение к упрощённому виду

Шаг 2. Вводим единичную матрицу

В линейной алгебре есть два вспомогательных понятия: обратная матрица и единичная матрица. Единичная матрица состоит из нулей, а по диагонали у неё единицы. Обратная матрица — это такая, которая при умножении на исходную даёт единичную матрицу.

Можно представить, что есть число 100 — это «сто в первой степени», 100 1

И есть число 0,01 — это «сто в минус первой степени», 100 -1

При перемножении этих двух чисел получится единица:
100 1 × 100 -1 = 100 × 0,01 = 1.

Вот такое, только в мире матриц.

Зная свойства единичных и обратных матриц, делаем алгебраическое колдунство. Умножаем обе известные матрицы на обратную матрицу А -1 . Неизвестную матрицу Х оставляем без изменений и переписываем уравнение:

А -1 × А × Х = А -1 × В

Добавляем единичную матрицу и упрощаем запись:

А -1 × А = E — единичная матрица

E × Х = А -1 × В — единичная матрица, умноженная на исходную матрицу, даёт исходную матрицу. Единичную матрицу убираем

Х = А -1 × В — новая запись уравнения

После введения единичной матрицы мы нашли способ выражения неизвестной матрицы X через известные матрицы A и B.

💡 Смотрите, что произошло: раньше нам нужно было найти неизвестную матрицу. А теперь мы точно знаем, как её найти: нужно рассчитать обратную матрицу A -1 и умножить её на известную матрицу B. И то и другое — замороченные процедуры, но с точки зрения арифметики — просто.

Шаг 3. Находим обратную матрицу

Вспоминаем формулу и порядок расчёта обратной матрицы:

  1. Делим единицу на определитель матрицы A.
  2. Считаем транспонированную матрицу алгебраических дополнений.
  3. Перемножаем значения и получаем нужную матрицу.

Собираем формулу и получаем обратную матрицу. Для удобства умышленно оставляем перед матрицей дробное число, чтобы было проще считать.

Третье действие: получаем обратную матрицу

Шаг 4. Вычисляем неизвестную матрицу

Нам остаётся посчитать матрицу X: умножаем обратную матрицу А -1 на матрицу B. Дробь держим за скобками и вносим в матрицу только при условии, что элементы новой матрицы будут кратны десяти — их можно умножить на дробь и получить целое число. Если кратных элементов не будет — дробь оставим за скобками.

Решаем матричное уравнение и находим неизвестную матрицу X. Мы получили кратные числа и внесли дробь в матрицу

Шаг 5. Проверяем уравнение

Мы решили матричное уравнение и получили красивый ответ с целыми числами. Выглядит правильно, но в случае с матрицами этого недостаточно. Чтобы проверить ответ, нам нужно вернуться к условию и умножить исходную матрицу A на матрицу X. В результате должна появиться матрица B. Если расчёты совпадут — мы всё сделали правильно. Если будут отличия — придётся решать заново.

👉 Часто начинающие математики пренебрегают финальной проверкой и считают её лишней тратой времени. Сегодня мы разобрали простое уравнение с двумя квадратными матрицами с четырьмя элементами в каждой. Когда элементов будет больше, в них легко запутаться и допустить ошибку.

Проверяем ответ и получаем матрицу B — наши расчёты верны

Ну и что

Алгоритм решения матричных уравнений несложный, если знать отдельные его компоненты. Дальше на основе этих компонентов математики переходят в более сложные пространства: работают с многомерными матрицами, решают более сложные уравнения, постепенно выходят на всё более и более абстрактные уровни. И дальше, в конце пути, появляется датасет из миллионов котиков. Этот датасет раскладывается на пиксели, каждый пиксель оцифровывается, цифры подставляются в матрицы, и уже огромный алгоритм в автоматическом режиме генерирует изображение нейрокотика:

Онлайн калькулятор. Решение систем линейных уравнений. Матричный метод. Метод обратной матрицы.

Используя этот онлайн калькулятор для решения систем линейных уравнений (СЛУ) матричным методом (методом обратной матрицы), вы сможете очень просто и быстро найти решение системы.

Воспользовавшись онлайн калькулятором для решения систем линейных уравнений матричным методом (методом обратной матрицы), вы получите детальное решение вашей задачи, которое позволит понять алгоритм решения задач на решения систем линейных уравнений, а также закрепить пройденный материал.

Решить систему линейных уравнений матричным методом

Изменить названия переменных в системе

Заполните систему линейных уравнений:

Ввод данных в калькулятор для решения систем линейных уравнений матричным методом

  • В онлайн калькулятор вводить можно числа или дроби. Более подробно читайте в правилах ввода чисел.
  • Для изменения в уравнении знаков с «+» на «-» вводите отрицательные числа.
  • Если в уравнение отсутствует какая-то переменная, то в соответствующем поле ввода калькулятора введите ноль.
  • Если в уравнение перед переменной отсутствуют числа, то в соответствующем поле ввода калькулятора введите единицу.

Например, линейное уравнение x 1 — 7 x 2 — x 4 = 2

будет вводится в калькулятор следующим образом:

Дополнительные возможности калькулятора для решения систем линейных уравнений матричным методом

  • Между полями для ввода можно перемещаться нажимая клавиши «влево», «вправо», «вверх» и «вниз» на клавиатуре.
  • Вместо x 1, x 2, . вы можете ввести свои названия переменных.

Вводить можно числа или дроби (-2.4, 5/7, . ). Более подробно читайте в правилах ввода чисел.

Решение матричных уравнений: теория и примеры

Решение матричных уравнений: как это делается

Матричные уравнения имеют прямую аналогию с простыми алгебраическими уравнениями, в которых присутствует операция умножения. Например,

где x — неизвестное.

А, поскольку мы уже умеем находить произведение матриц, то можем приступать к рассмотрению аналогичных уравнений с матрицами, в которых буквы — это матрицы.

Итак, матричным уравнением называется уравнение вида

где A и B — известные матрицы, X — неизвестная матрица, которую требуется найти.

Как решить матричное уравнение в первом случае? Для того, чтобы решить матричное уравнение вида AX = B , обе его части следует умножить на обратную к A матрицу слева:

.

По определению обратной матрицы, произведение обратной матрицы на данную исходную матрицу равно единичной матрице: , поэтому

.

Так как E — единичная матрица, то EX = X . В результате получим, что неизвестная матрица X равна произведению матрицы, обратной к матрице A , слева, на матрицу B :

.

Как решить матричное уравнение во втором случае? Если дано уравнение

то есть такое, в котором в произведении неизвестной матрицы X и известной матрицы A матрица A находится справа, то нужно действовать аналогично, но меняя направление умножения на матрицу, обратную матрице A , и умножать матрицу B на неё справа:

,

,

.

Как видим, очень важно, с какой стороны умножать на обратную матрицу, так как . Обратная к A матрица умножается на матрицу B с той стороны, с которой матрица A умножается на неизвестную матрицу X . То есть с той стороны, где в произведении с неизвестной матрицей находится матрица A .

Как решить матричное уравнение в третьем случае? Встречаются случаи, когда в левой части уравнения неизвестная матрица X находится в середине произведения трёх матриц. Тогда известную матрицу из правой части уравнения следует умножить слева на матрицу, обратную той, которая в упомянутом выше произведении трёх матриц была слева, и справа на матрицу, обратную той матрице, которая располагалась справа. Таким образом, решением матричного уравнения

.

Решение матричных уравнений: примеры

Пример 1. Решить матричное уравнение

.

Решение. Данное уравнение имеет вид AX = B , то есть в произведении матрицы A и неизвестной матрицы X матрица A находится слева. Поэтому решение следует искать в виде , то есть неизвестная матрица равна произведению матрицы B на матрицу, обратную матрице A слева. Найдём матрицу, обратную матрице A .

Сначала найдём определитель матрицы A :

.

Найдём алгебраические дополнения матрицы A :

.

Составим матрицу алгебраических дополнений:

.

Транспонируя матрицу алгебраических дополнений, находим матрицу, союзную с матрицей A :

.

Теперь у нас есть всё, чтобы найти матрицу, обратную матрице A :

.

Наконец, находим неизвестную матрицу:

Пример 2. Решить матричное уравнение

.

Пример 3. Решить матричное уравнение

.

Решение. Данное уравнение имеет вид XA = B , то есть в произведении матрицы A и неизвестной матрицы X матрица A находится справа. Поэтому решение следует искать в виде , то есть неизвестная матрица равна произведению матрицы B на матрицу, обратную матрице A справа. Найдём матрицу, обратную матрице A .

Сначала найдём определитель матрицы A :

.

Найдём алгебраические дополнения матрицы A :

.

Составим матрицу алгебраических дополнений:

.

Транспонируя матрицу алгебраических дополнений, находим матрицу, союзную с матрицей A :

.

Находим матрицу, обратную матрице A :

.

Находим неизвестную матрицу:

До сих пор мы решали уравнения с матрицами второго порядка, а теперь настала очередь матриц третьего порядка.

Пример 4. Решить матричное уравнение

.

Решение. Это уравнение первого вида: AX = B , то есть в произведении матрицы A и неизвестной матрицы X матрица A находится слева. Поэтому решение следует искать в виде , то есть неизвестная матрица равна произведению матрицы B на матрицу, обратную матрице A слева. Найдём матрицу, обратную матрице A .

Сначала найдём определитель матрицы A :

.

Найдём алгебраические дополнения матрицы A :

Составим матрицу алгебраических дополнений:

Транспонируя матрицу алгебраических дополнений, находим матрицу, союзную с матрицей A :

.

Находим матрицу, обратную матрице A , и делаем это легко, так как определитель матрицы A равен единице:

.

Находим неизвестную матрицу:

Пример 5. Решить матричное уравнение

.

Решение. Данное уравнение имеет вид XA = B , то есть в произведении матрицы A и неизвестной матрицы X матрица A находится справа. Поэтому решение следует искать в виде , то есть неизвестная матрица равна произведению матрицы B на матрицу, обратную матрице A справа. Найдём матрицу, обратную матрице A .

Сначала найдём определитель матрицы A :

.

Найдём алгебраические дополнения матрицы A :

Составим матрицу алгебраических дополнений:

.

Транспонируя матрицу алгебраических дополнений, находим матрицу, союзную с матрицей A :

.

Находим матрицу, обратную матрице A :

.

Находим неизвестную матрицу:

Пример 6. Решить матричное уравнение

.

Решение. Данное уравнение имеет вид AXB = C , то есть неизвестная матрица X находится в середине произведения трёх матриц. Поэтому решение следует искать в виде . Найдём матрицу, обратную матрице A .

Сначала найдём определитель матрицы A :

.

Найдём алгебраические дополнения матрицы A :

.

Составим матрицу алгебраических дополнений:

.

Транспонируя матрицу алгебраических дополнений, находим матрицу, союзную с матрицей A :

.

Находим матрицу, обратную матрице A :

.

Найдём матрицу, обратную матрице B .

Сначала найдём определитель матрицы B :

.

Найдём алгебраические дополнения матрицы B :

Составим матрицу алгебраических дополнений матрицы B :

.

Транспонируя матрицу алгебраических дополнений, находим матрицу, союзную с матрицей B :

.

Находим матрицу, обратную матрице B :

.

источники:

http://ru.onlinemschool.com/math/assistance/equation/matr/

http://function-x.ru/matrix_equations.html

Определение:
Обратная матрица — такая матрица , при умножении на которую, исходная матрица даёт в результате единичную матрицу

Содержание

  • 1 Обратимость в алгебре
  • 2 Критерий обратимости матрицы
  • 3 Свойства обратной матрицы
  • 4 Методы нахождения обратной матрицы
    • 4.1 Метод Гаусса для нахождения обратной матрицы
      • 4.1.1 Пример
    • 4.2 Метод присоединенной матрицы
      • 4.2.1 Алгоритм получения обратной матрицы
  • 5 Ссылки
  • 6 Источники

Обратимость в алгебре

Определение:
Пусть — алгебра над . называется единицей , если , причем единственна
Определение:
Пусть в алгебре , тогда называется левым обратным по отношению к , а — правым обратным по отношению к
Определение:
Пусть . Левый обратный элементу , являющийся одновременно и правым обратным к нему, называется обратным и обозначается . При этом сам элемент называется обратимым.
Лемма:

Пусть алгебре

— левый обратный

— правый обратный.

Тогда обратим, при этом и

Доказательство:

Факт 1. , но , тогда по определению .

Факт 2. Пусть

, но

Критерий обратимости матрицы

Теорема:

Квадратная матрица обратима (имеет обратную матрицу) тогда и только тогда, когда она невырожденная, то есть .

Доказательство:

Шаг 1. Если матрица обратима, то для некоторой матрицы . Тогда, если квадратные матрицы одного и того же порядка, то :

, следовательно, .

Шаг 2. Докажем обратное утверждение. Пусть .

1) Докажем существование правой обратной матрицы .

Предположим , где

, фиксируем , тогда:

, тогда получим, что — матрица системы уравнений, так как , то по Крамеру

В итоге для всех получим матрицу , что и требовалось.

2) Докажем существование левой обратной матрицы .

Предположим

Фиксируем , тогда ,получаем заполнение по строчкам, аналогично первому пункту показываем .

3) Тогда по лемме , теорема доказана.

Свойства обратной матрицы

Методы нахождения обратной матрицы

Метод Гаусса для нахождения обратной матрицы

Возьмём две матрицы: саму и . Приведём матрицу к единичной матрице методом Гаусса. После применения каждой операции к первой матрице применим ту же операцию ко второй. Когда приведение первой матрицы к единичному виду будет завершено, вторая матрица окажется равной .

Пример

Найдем обратную матрицу для матрицы

  • 1) Для начала убедимся, что ее определитель не равен нулю(она невырожденная).
  • 2) Справа от исходной матрицы припишем единичную.
  • 3) Методом Гаусса приведем левую матрицу к единичной, применяя все операции одновременно и к левой, и к правой матрицам.
  • 4)

Метод присоединенной матрицы

, где  — присоединенная матрица;
Определение:
Присоединенная(союзная, взаимная) матрица — матрица, составленная из алгебраических дополнений для соответствующих элементов исходной матрицы.

Исходная матрица:

Где:

  • — присоединённая(союзная, взаимная) матрица;
  • — алгебраические дополнения исходной матрицы;
  • — элементы исходной матрицы.

Алгебраическим дополнением элемента матрицы называется число

,

где  — дополнительный минор, определитель матрицы, получающейся из исходной матрицы путем вычёркивания i -й строки и j -го столбца.

Алгоритм получения обратной матрицы

  • заменить каждый элемент исходной матрицы на его алгебраическое дополнение — в результате будет получена присоединенная матрица
  • разделить каждый элемент транспонированной присоединенной матрицы на определитель исходной матрицы.

Ссылки

  • Обратный оператор

Источники

  • Википедия — Обратная матрица
  • Википедия — Союзная матрица
  • Википедия — Алгебраическое дополнение
  • Wikipedia — Invertible matrix
  • Анин конспект

Понравилась статья? Поделить с друзьями:
  • Как найти все песни с одним минусом
  • Созвездие водолея как его найти
  • Как найти пишу в лесу
  • Right before trigger this is the real bug setup checkra1n как исправить
  • Ошибка в табеле как исправить в зуп 1с